Крок 1 - Стоматологія 2012 (буклет)

1 / 200
Гіпоплазія емалі зумовлена домінантним геном, локалізованим в Х-хромосомі. Мати має нормальну емаль зубів, а у батька спостерігається гіпоплазія емалі. У кого з дітей буде виявлятися ця аномалія? Enamel hypoplasia is caused by a dominant gene located on the X chromosome. The mother has normal tooth enamel, and the father has enamel hypoplasia. Which of the children will have this anomaly ?

У всіх дітей All children have

Тільки у дочок Only in daughters

Тільки у синів Only sons

У половини синів Half sons

У половини дочок Half daughters

2 / 200
У районах Південної Африки у людей розповсюджена серпоподібно-клітинна анемія, при якій еритроцити мають форму серпа внаслідок зміни в молекулі гемоглобіну амінокислоти глутаміну на валін. Причиною цієї хвороби є: In areas of South Africa, sickle cell anemia is widespread among people, in which red blood cells have a sickle shape due to a change in the hemoglobin molecule from the amino acid glutamine to valine. The cause of this disease is:

Порушення механізмів реалізації генетичної інформації Violation of genetic information implementation mechanisms

Трансдукція Transduction

Кросинговер Crossover

Генна мутація Gene mutation

Геномні мутації Genomic mutations

3 / 200
При обстеженні дівчини 18-ти років знайдені наступні ознаки: недорозвинення яєчників, широкі плечі, вузький таз, вкорочення нижніх кінцівок, 'шия сфінкса', розумовий розвиток не порушено. Встановлено діагноз: синдром Шерешевського-Тернера. Яке хромосомне порушення у хворої? During the examination of an 18-year-old girl, the following signs were found: underdevelopment of the ovaries, broad shoulders, narrow pelvis, shortening of the lower limbs, 'neck of the sphinx', mental development is not impaired The diagnosis was established: Shereshevsky-Turner syndrome. What chromosomal disorder does the patient have?

Нульсомія Х Nulsomia X

Трисомія 13 Trisomy 13

Моносомія Х Monosomy X

Трисомія Х Trisomy X

Трисомія 18 Trisomy 18

4 / 200
У 5-річної дівчинки на періанальних складках мати знайшла білих 'черв’ячків' , які викликають у дитини свербіж і неспокій, і доставила їх до лабораторії. При дослідженні лікар побачив білих гельмінтів 0,5-1 см довжиною, ниткоподібної форми з загостреними кінцями, у деяких вони заокруглені. Який діагноз можна встановити? In a 5-year-old girl, the mother found white 'worms' on the perianal folds, which cause itching and restlessness in the child, and took them to the laboratory. During the examination the doctor saw white helminths 0.5-1 cm long, thread-like in shape with pointed ends, in some they are rounded. What diagnosis can be made?

Теніоз Taeniosis

Дифілоботріоз Diphyllobotriosis

Опісторхоз Opistorchosis

Ентеробіоз Enterobiosis

Аскаридоз Ascariasis

5 / 200
У дитини періодично з’являються рідкі випорожнення, іноді біль у ділянці живота, нудота, блювання. Зі слів матері, одного разу у дитини з блювотною масою виділився гельмінт веретеноподібної форми, розміром 20 см. Причиною такого стану може бути: The child periodically has loose stools, sometimes pain in the abdomen, nausea, vomiting. According to the mother, once the child with vomiting mass had a spindle worm shape, size 20 cm. The reason for this condition can be:

Трихінельоз Trichinellosis

Трихоцефальоз Trichocephalus

Дракункульоз Dracunculosis

Аскаридоз Ascariasis

Анкілостомоз Ankylostomosis

6 / 200
У 60-річної жінки з тяжкою формою парадонтозу при мікроскопічному дослідженні зіскобу з ясен були виявлені одноядерні найпростіші розміром 360 мкм з широкими псевдоподіями. Які найпростіші були виявлені у хворої? In a 60-year-old woman with a severe form of periodontitis, microscopic examination of a scraping from the gums revealed mononuclear protozoa measuring 360 μm with wide pseudopodia. What protozoa were found in the patient?

Balantidium coli Balantidium coli

Trichomonas tenax Trichomonas tenax

Toxoplasma gondii Toxoplasma gondii

Entamoeba gingivalis Entamoeba gingivalis

Entamoeba histolytica Entamoeba histolytica

7 / 200
У хворого внаслідок неадекватного лікування запалення середнього вуха (мезотимпаніт) виникло гнійне запалення комірок соскоподібного відростка скроневої кістки (мастоїдит). Гнійник прорвався у ложе грудино-ключично-соскоподібного м’яза. У межах якої фасції шиї локалізується патологічний процес? As a result of inadequate treatment of inflammation of the middle ear (mesotympanitis), the patient developed purulent inflammation of the cells of the mastoid process of the temporal bone (mastoiditis). The abscess broke into the bed of the sternoclavicular-mastoid m 'Yaz. Within which fascia of the neck is the pathological process localized?

Поверхнева фасція Superficial Fascia

Поверхневий листок власної фасції Superficial sheet of own fascia

Глибокий листок власної фасції Deep sheet of own fascia

Внутрішня фасція Internal Fascia

Передхребетна фасція Prevertebral fascia

8 / 200
Після видалення II малого кутнього зуба верхньої щелепи у хворого відзначається коміркова кровотеча. Пошкодженням якої артерії воно викликане? After the removal of the II small canine tooth of the upper jaw, the patient has a cellular hemorrhage. What artery is it damaged?

Нижня альвеолярна Inferior alveolar

Задня верхня альвеолярна Posterior superior alveolar

Передня верхня альвеолярна Anterior superior alveolar

Піднебінна Palate

Середня верхня альвеолярна Medium Upper Alveolar

9 / 200
Чоловік 30-ти років звернувся до стоматолога зі скаргою на розлади жування, виникнення болю при відтягуванні щелепи назад. Лікар встановив запалення одного з жувальних м’язів. Якого саме? A 30-year-old man went to the dentist with a complaint of chewing disorders, pain when pulling the jaw back. The doctor diagnosed inflammation of one of the chewing muscles. What exactly ?

Крилоподібний латеральний Winged Lateral

Скроневий (передні волокна) Temporal (anterior fibers)

Крилоподібний медіальний Prioform medial

Жувальний Chewing

Скроневий (задні волокна) Temporal (posterior fibers)

10 / 200
Юнака 18-ти років доставлено в лікарню з ознаками внутрішньої кровотечі. Під час гри в футбол він отримав удар в ділянку лівого підребер’я. Ушкодження якого органу спричинило сильну кровотечу? An 18-year-old boy was taken to the hospital with signs of internal bleeding. While playing football, he was hit in the left hypochondrium. Damage to which organ caused a strong bleeding?

Fundus ventriculi Fundus ventriculi

Ren sinistra Ren sinistra

Cauda pancreatis Cauda pancreatis

Lien Lien

Flexura coli sinistra Flexura coli sinistra

11 / 200
У дітей часто можна спостерігати утруднене носове дихання, яке пов’язане з надмірним розвитком лімфої-дної тканини слизової оболонки глотки. Розростання яких мигдаликів може спричинити це явище? Difficult nasal breathing can often be observed in children, which is associated with excessive development of the lymphoid tissue of the mucous membrane of the pharynx. The growth of which tonsils can cause this phenomenon?

Tonsilla tubaria Tonsilla tubaria

Tonsilla palatina Tonsilla palatina

Tonsilla pharyngea Tonsilla pharyngea

Tonsilla lingualis Tonsilla lingualis

Усіх названих мигдаликів All named tonsils

12 / 200
У хворого 26-ти років виявлений великий фурункул м’яких тканин обличчя біля кореню носа та нижньої повіки. Грізним ускладненням цього захворювання може бути розповсюдження інфекції по венозним сполученням цього регіону до пазух твердої мозкової оболонки. Яка з пазух найбільш імовірно може бути уражена? A 26-year-old patient was diagnosed with a large furuncle of the soft tissues of the face near the root of the nose and the lower eyelid. A formidable complication of this disease can be the spread of infection through the venous connections of this region to the sinuses of the dura mater. Which of the sinuses is most likely to be affected?

Печериста Cave

Сигмоподібна Sigma

Потилична Occipital

Верхня сагітальна Upper sagittal

Верхня кам’яниста Upper stony

13 / 200
У хворого з флегмоною щоки розвинувся менінгіт. Поширення інфекції, найбільш імовірно, відбулося по анастомозу між: A patient with phlegmon of the cheek developed meningitis. The infection most likely spread through an anastomosis between:

Скроневою і надочноямковою артеріями Temporal and supraorbital arteries

Лицевою і позадущелепною венами Facial and extramaxillary veins

Лицевою і очною артеріями Facial and eye arteries

Лицевою веною і венами очної ямки Facial and orbital veins

Лицевою веною та крилоподібним сплетінням Facial vein and pterygoid plexus

14 / 200
У потерпілого лівобічний уламковий перелом виличної кістки з втратою шкірної чутливості над нею. Який нерв пошкоджений? The victim has a left-sided fragmentary fracture of the zygomatic bone with loss of skin sensitivity above it. Which nerve is damaged?

Вилично-лицевий Bulk-facial

Щічний Buccal

Нижньоочноямковий Inferior foveal

Мала гусяча лапка Small crow's foot

Лицевий Face

15 / 200
Хворий з запаленням слизової язика скаржиться на розлад загальної чутливості передніх двох третин язика. Ураженням якого нерва це викликано? A patient with inflammation of the mucous membrane of the tongue complains of a disorder of the general sensitivity of the front two thirds of the tongue. What nerve is this caused by?

Барабанна струна Drum String

Язикоглотковий Lingopharyngeal

Язиковий Language

Блукаючий Wandering

Барабанний Drum

16 / 200
У хворого 25-ти років, який скаржився на утруднене ковтання, виявлене пухлиноподібне підвищення на корені язика 1-2 см в діаметрі, в ділянці сліпого отвору. Розростання залишків якої залози встановив лікар? A 25-year-old patient who complained of difficulty swallowing was found to have a tumor-like growth 1-2 cm in diameter at the root of the tongue, in the area of the blind hole. Residual growth which gland did the doctor install?

Прищитоподібна Pythroid

Під’язикова Sublingual

Аденогіпофіз Adenohypophysis

Щитоподібна Thyroid

Вилочкова Vylochkova

17 / 200
У хворого внаслідок пошкодження шкіри в середній ділянці грудинно-ключичнососкоподібного м’яза виникла повітряна емболія. Яка вена шиї була травмована? The patient had an air embolism due to damage to the skin in the middle part of the sternocleidomastoid muscle. Which neck vein was injured?

Поперечна вена шиї Transverse jugular vein

Внутрішня яремна Internal jugular

Передня яремна Anterior jugular

Задня вушна Back auricle

Зовнішня яремна External jugular

18 / 200
Під час дослідження обличчя хворого відзначається, що правий кут рота зміщений у лівий бік, права щока і губи притиснені до зубів і ясен. Порушенням функції якого м’яза це викликане? During the examination of the patient's face, it is noted that the right corner of the mouth is shifted to the left side, the right cheek and lips are pressed against the teeth and gums. A violation of the function of which muscle is this caused?

Коловий м’яз рота Orbital muscle

М’яз, що опускає кут рота Muscle that lowers the corner of the mouth

Виличні м’язи Chelic muscles

М’яз сміху Smiling muscle

Щічний м’яз Buccal muscle

19 / 200
До лікаря звернувся хворий з запаленням комірок решітчастої кістки (етмоїдит). При обстеженні виявлено порушення кровопостачання кістки. Гілками якої артерії кровопостачаю-ться решітчасті комірки в нормі? A patient with inflammation of the cells of the ethmoid bone (ethmoiditis) consulted a doctor. During the examination, a violation of the blood supply to the bone was revealed. Which artery's branches normally supply blood to the ethmoid cells?

A. ophthalmica A. ophthalmica

A. facialis A. facialis

A. transversa faciei A. transversa faciei

A. infraorbitalis A. infraorbitalis

A. cerebri anterior A. cerebri anterior

20 / 200
У пораненого кровотеча із гілок сонної артерії. Для тимчасової зупинки кровотечі сонну артерію треба притиснути до горбика поперечного відростка шийного хребця. Якого саме? The wounded person is bleeding from the branches of the carotid artery. To temporarily stop the bleeding, the carotid artery should be pressed against the tubercle of the transverse process of the cervical vertebra. Which one exactly?

II II

III III

VI VI

IV IV

V V

21 / 200
У чоловіка 40-ка років запалення яєчка ускладнилось його водянкою. Необхідне оперативне втручання. Яку з оболонок яєчка останньою розтинає хірург під час операції? In a 40-year-old man, inflammation of the testicle was complicated by his dropsy. Surgical intervention is necessary. Which of the testicle membranes is the last to be dissected by the surgeon during the operation?

Зовнішня сім’яна фасція External spermatic fascia

М ’ясиста оболонка Fleshy shell

Внутрішня сім’яна фасція Internal spermatic fascia

М’яз-підіймач яєчка Elevator testis muscle

Парієтальний листок піхвової оболонки яєчка Parietal sheet of the vaginal membrane of the testis

22 / 200
Жінка 52-х років після травми. При огляді констатовано зниження пульсації, блідість шкіри і зниження температури в ділянці великого пальця правої руки. Ушкодження якої з артерій верхньої кінцівки, найімовірніше, приводить до таких наслідків? A 52-year-old woman after an injury. During the examination, a decrease in pulsation, pallor of the skin, and a decrease in temperature in the area of the thumb of the right hand were noted. Damage to the arteries of the upper limb, most likely, leads to such consequences?

A.princeps pollicis A.princeps pollicis

A.radialis A.radialis

A.digitalis communis A.digitalis communis

A.ulnaris A.ulnaris

A.digitalis propria A.digitalis propria

23 / 200
У хворої з пухлиною підшлункової залози розвинулася механічна жовтяниця внаслідок стиснення жовчовивідної протоки. Яка протока піддається стисненню? A patient with a pancreatic tumor developed mechanical jaundice as a result of compression of the bile duct. Which duct is subject to compression?

Ductus choledochus Ductus choledochus

Ductus hepaticus sinister Ductus hepaticus sinister

Ductus hepaticus dexter Ductus hepaticus dexter

Ductus hepaticus communis Ductus hepaticus communis

Ductus cysticus Ductus cysticus

24 / 200
В гістопрепараті представлена частина органу, що містить багатошаровий плоский незроговілий епітелій та нижче розташовані сполучнотканинні пластинки, які не містять судин. Який це орган? In the histopreparation, a part of an organ is presented, which contains a multi-layered flat non-keratinized epithelium and connective tissue plates that do not contain blood vessels are located below. What kind of organ is this?

Кришталик Crystal

Рогівка Cornea

Сітківка Retina

Слизова оболонка стравоходу Esophageal mucosa

Слизова оболонка ротової порожнини Mucosa of the oral cavity

25 / 200
У гістологічному препараті шліфа зуба у міжклітинній речовині визначаються колагенові волокна, що йдуть тангенційно до дентино-емалевої межі і перпендикулярно до дентинних трубочок (волокна Ебнера). Назвіть даний шар дентину: In the histological preparation of a tooth slide, collagen fibers are identified in the intercellular substance, which go tangentially to the dentin-enamel border and perpendicular to the dentin tubules (Ebner's fibers). Name this layer dentine:

Навколопульпарний дентин Peripulpal dentine

Плащовий дентин Coat dentin

Вторинний дентин Secondary dentine

Зернистий шар Grain layer

Інтерглобулярний дентин Interglobular dentine

26 / 200
У культурі тканин ядерним опроміненням пошкоджені ядерця ядер. Відновлення яких органел у цитоплазмі клітин стає проблематичним? In tissue culture, nuclear irradiation damaged the nucleoli of the nuclei. The restoration of which organelles in the cytoplasm of cells becomes problematic?

Комплекс Іольджі Iolji Complex

Лізосоми Lysosomes

Мікротрубочки Microtubules

Рибосоми Ribosomes

Ендоплазматична сітка Endoplasmic reticulum

27 / 200
Під час гаструляції у зародку недостатньо сформувався первинний Іензе-новський вузлик. Розвиток якого осьового органу загальмується? During gastrulation, the embryo did not sufficiently form the primary Ienzen nodule. The development of which axial organ is inhibited?

Нервові гребінці Nerve combs

Нервовий жолобок Neural Groove

Нервова трубка Neural tube

Мантійний шар нервової трубки Neural tube mantle layer

Хорда Horde

28 / 200
У дитини на шкірі навколо подряпини виникли ознаки запалення (біль, почервоніння та набряк), як ознаки негайної гіперчутливості. Які клітини крові обумовлюють ці зміни? The child has signs of inflammation (pain, redness, and swelling) on the skin around the scratch, as signs of immediate hypersensitivity. What blood cells cause these changes?

Моноцити Monocytes

Нейтрофіли Neutrophils

Еозинофіли Eosinophils

Лімфоцити Lymphocytes

Базофіли Basophils

29 / 200
В гістологічному препараті відділ нервової системи, в якому визначається пошарове розташування нейроци-тів, серед яких є клітини таких форм: зірчасті, веретеноподібні, горизонтальні, пірамідні. Який це відділ нервової системи? In a histological preparation, a section of the nervous system in which the layer-by-layer arrangement of neurocytes is determined, among which there are cells of the following shapes: star-shaped, spindle-shaped, horizontal, pyramidal. What is this department of the nervous system?

Вегетативний вузол Vegetative node

Кора великих півкуль головного мозку Cortex of the cerebral hemispheres

Спинний мозок Spinal cord

Кора мозочка Cerebellar cortex

Спинномозковий вузол Spinal node

30 / 200
У крові дівчини 16-ти років, котра страждає на аутоiмунне запалення щи-топоді6ної залози, виявлено числєнні плазматичні клітини. З проліферацією та диференціюванням яких клітин крові пов’язано збільшення кількості пла-змоцитів? Numerous plasma cells were found in the blood of a 16-year-old girl suffering from autoimmune inflammation of the thyroid gland. The proliferation and differentiation of which blood cells is associated with increasing the number of plasma cells?

В-лімфоцитів B-lymphocytes

Т-супресорів T-suppressors

Тканинних базофілів Tissue basophils

Т-кілерів T-killers

Т-хелперів T-helpers

31 / 200
В препараті діагностується тканина, в якій клітини розміщуються поодинці та ізогрупами, а в міжклітинній речовині не видно волокнистих структур. Яка тканина присутня в препараті? A tissue is diagnosed in the preparation, in which cells are located singly and in isogroups, and fibrous structures are not visible in the intercellular substance. What kind of tissue is present in the preparation?

Гладенька м’язова Smooth muscular

Кісткова Bone

Гіалінова хрящова Hyaline cartilage

Епітеліальна Epithelial

Волокниста хрящова Fibrocartilage

32 / 200
При дослідженні оболонок головного мозку медичний експерт знайшов зяючі венозні судини, що зрощені з навколишніми тканинами. Назвіть, до яких вен відносяться ці судини: During the examination of the membranes of the brain, the medical expert found gaping venous vessels fused with the surrounding tissues. Name which veins these vessels belong to:

Вени безм’язового типу Veins of muscleless type

Вени із середнім розвитком м’язових елементів Veins with average development of muscular elements

Венули Venules

Вени із сильним розвитком м’язових елементів Veins with strong development of muscular elements

Вени із слабким розвитком м’язових елементів Veins with weak development of muscular elements

33 / 200
В гострому досліді тварині в порожнину 12-ти палої кишки ввели слабкий розчин хлористоводневої кислоти. До збільшення секреції якого гастроін-тестинального гормону це призведе? In an acute experiment, a weak solution of hydrochloric acid was injected into the duodenal cavity of an animal. This will lead to an increase in the secretion of which gastrointestinal hormone?

Мотилін Motilin

Секретин Secretin

Гістамін Histamine

Нейротензин Neurotensin

Гастрин Gastrin

34 / 200
При визначенні групової належності крові за системою АВ0 аглютина- цію еритроцитів досліджуваної крові викликали стандартні сироватки першої та другої груп і не викликала -третьої групи. Якої група досліджувана кров? When determining the blood group according to the AB0 system, agglutination of erythrocytes of the examined blood was caused by standard serums of the first and second groups and not caused by the third group. What group is the examined blood?

a(ii) в a(ii) in

B (III) а B (III) а

- -

0 (I) а,в 0 (I) a,c

АВ (IV) AB (IV)

35 / 200
У хворого з пересадженим серцем при фізичному навантаженні збільшився хвилинний об’єм крові. Який механізм регуляції забезпечує ці зміни? The minute volume of blood increased during physical exertion in a patient with a transplanted heart. What regulation mechanism ensures these changes?

Катехоламіни Catecholamines

Парасимпатичні умовні рефлекси Parasympathetic conditioned reflexes

Парасимпатичні безумовні рефлекси Parasympathetic unconditioned reflexes

Симпатичні безумовні рефлекси Sympathetic unconditioned reflexes

Симпатичні умовні рефлекси Sympathetic conditioned reflexes

36 / 200
При аналізі ЕКГ людини з’ясовано, що у другому стандартному відведенні від кінцівок зубці T позитивні, їх амплітуда та тривалість у нормі. Вірним є висновок, що у шлуночках серця нормально відбувається процес: When analyzing a human ECG, it was found that in the second standard lead from the limbs T waves are positive, their amplitude and duration are normal. The conclusion is correct that in the ventricles the process of the heart is normal:

Реполяризації Repolarizations

Деполяризації Depolarizations

Збудження Arousal

Розслаблення Relax

Скорочення Abbreviation

37 / 200
У хворого виявлено різке зниження активності сурфактанту легень. Що буде наслідком цього? A sharp decrease in lung surfactant activity was detected in the patient. What will be the consequence of this?

Зменшення роботи дихальних м’язів Decreased work of respiratory muscles

Збільшення вентиляції легень Increasing lung ventilation

Зменшення опору дихальних шляхів Reduced airway resistance

Схильність альвеол до спадання Tendency of alveoli to decline

Гіпероксемія Hyperoxemia

38 / 200
В результаті черепно-мозкової травми у хворого були виявлені наступні ознаки: інтенційний тремор, дисме-трія, адіадохокінез, дизартрія. Яка структура головного мозку пошкоджена? As a result of a craniocerebral injury, the patient had the following symptoms: intention tremor, dysmetria, adiadochokinesis, dysarthria. What brain structure is damaged?

Мозочок Cerebellum

Бліда куля Pale Orb

Рухова кора Motor cortex

Чорна речовина Black Matter

Стріатум Striatum

39 / 200
У хворого 70-ти років діагностовано крововилив у стовбур мозку. Обстеження виявило підвищення тонусу м’язів-згиначів на тлі зниження тонусу м’язів-розгиначів. Подразненням яких структур мозку можна пояснити зміни у тонусі м’язів? A 70-year-old patient was diagnosed with a brain stem hemorrhage. The examination revealed an increase in the tone of the flexor muscles on the background of a decrease in the tone of the extensor muscles. The irritation of which brain structures can explain changes in muscle tone?

Червоні ядра Red kernels

Ретикулярна формація Reticular Formation

Чорна речовина Black Matter

Вестибулярні ядра Vestibular nuclei

Чотиригорбикова структура Four hump structure

40 / 200
У пацієнта після повторного протезування зу6ів виникли сухість і металічний присмак у роті, спотворення смаку, запалення слизової оболонки язика і ясен. Найбільш імовірною причиною вказаних проявів є: After re-prosthetic teeth, the patient developed dryness and a metallic taste in the mouth, taste distortion, inflammation of the mucous membrane of the tongue and gums. The most likely cause of these symptoms is:

Занесення інфекції Infection

Пошкодження смакових рецепторів Damage to taste buds

Застосування неякісної пластмаси Use of low-quality plastic

Пошкодження чутливих нервових волокон Damage of sensitive nerve fibers

Явище гальванізму Galvanism phenomenon

41 / 200
При обстежені пацієнта встановили що він має сильний, врівноважений, інертний тип вищої нервової діяльності за Павловим. Який темперамент за Гіппократом має цей пацієнт? When examining the patient, it was established that he has a strong, balanced, inert type of higher nervous activity according to Pavlov. What temperament does this patient have according to Hippocrates?

Холеричний Choleric

Сангвінічний Sanguine

Флегматичний Phlegmatic

- -

Меланхолічний Melancholic

42 / 200
У жінки 32-х років запалення ясен супроводжується їх гіпоксією. Утворення якого метаболіту вуглеводного обміну значно збільшується при цьому в тканинах пародонта? In a 32-year-old woman, inflammation of the gums is accompanied by their hypoxia. The formation of which metabolite of carbohydrate metabolism significantly increases in this case in the periodontal tissues?

Лактат Lactate

Глікоген Glycogen

НАДФ-Н NADF-H

Рибозо-5-фосфат Ribose-5-phosphate

Ілюкозо-6-фосфат Ilucose-6-phosphate

43 / 200
У хворого з нирковою недостатністю розвинулась остеодистрофія, що супроводжується інтенсивною деміне-ралізацією кісток. Порушення утворення активної форми якого вітаміну є причиною даного ускладнення? A patient with kidney failure developed osteodystrophy, which is accompanied by intense bone demineralization. Violation of the formation of the active form of which vitamin is the cause of this complication?

Рибофлавін Riboflavin

Кальциферол Calciferol

Тіамін Thiamine

Ретинол Retinol

Нафтохінон Naphtoquinone

44 / 200
При обробці перекисом водню слизової оболонки ротової порожнини хворого, кров пофарбувалась у кори- чневий колір замість піноутворення. При зниженні концентрації якого з перелічених ферментів це можливо? When treating the patient's oral mucosa with hydrogen peroxide, the blood turned brown instead of foaming. When the concentration of which of the listed enzymes is reduced, this is possible?

Псевдохолінестераза Pseudocholinesterase

Ацетилтрансфераза Acetyltransferase

Метгемоглобінредуктаза Methemoglobin reductase

Ілюкозо-6-фосфатдегідрогеназа Ilucose-6-phosphate dehydrogenase

Каталаза Catalase

45 / 200
При цукровому діабеті і голодуванні в крові збільшується вміст ацетонових тіл, що використовуються в якості енергетичного матеріалу. Назвіть речовину, з якої вони синтезуються: With diabetes and starvation, the content of acetone bodies, which are used as energy material, increases in the blood. Name the substance from which they are synthesized:

Малат Malat

Цитрат Citrate

Сукциніл-КоА Succinyl-CoA

Кетоглутарат Ketoglutarate

Ацетіл-КоА Acetyl-CoA

46 / 200
При обстеженні хворого виявлені дерматит, діарея, деменція. Відсутність якого вітаміну є причиною цього стану? During the examination of the patient, dermatitis, diarrhea, dementia were found. The lack of which vitamin is the cause of this condition?

Біотин Biotin

Фолієва кислота Folic acid

Нікотинамід Nicotinamide

Рутин Routine

Аскорбінова кислота Ascorbic acid

47 / 200
Електрофоретичне дослідження сироватки крові хворого на пневмонію показало збільшення одної з білкових фракцій. Вкажіть її: Electrophoretic examination of the blood serum of a pneumonia patient showed an increase in one of the protein fractions. Specify it:

y-глобуліни y-globulins

Альбуміни Albumins

в-глобуліни b-globulins

а2-глобуліни a2-globulins

аі-глобуліни ai-globulins

48 / 200
Мати помітила темну сечу у її 5-річної дитини. Жовчних пігментів у сечі не виявлено. Встановлено діагноз: ал-каптонурія. Дефіцит якого ферменту має місце? A mother noticed dark urine in her 5-year-old child. Bile pigments were not detected in the urine. The diagnosis was made: al-captonuria. Which enzyme is deficient?

Тирозиназа Tyrosinase

Декарбоксилаза фенілпірувату Phenylpyruvate decarboxylase

Оксидаза оксифенілпірувату Oxyphenylpyruvate oxidase

Оксидаза гомогентизинової кислоти Homogentisic acid oxidase

Фенілаланінгідроксилаза Phenylalanine hydroxylase

49 / 200
У хворого в крові збільшена концентрація пірувату. Значна кількість його екскретується з сечею. Який авітаміноз спостерігається у хворого? The patient has an increased concentration of pyruvate in the blood. A significant amount of it is excreted in the urine. What type of vitamin deficiency is observed in the patient?

B2 B2

B1 B1

B3 B3

E E

B6 B6

50 / 200
У чоловіка 42-х років, який страждає на подагру, в крові підвищена концентрація сечової кислоти. Для зниження рівня сечової кислоті йому призначено аллопуринол. Конкурентним інгібітором якого ферменту є аллопу-ринол? A 42-year-old man suffering from gout has an increased concentration of uric acid in his blood. To reduce the level of uric acid, he is prescribed allopurinol. A competitive inhibitor of which enzyme is allopurinol?

Гіпоксантинф осф орибозилтрансфе-раза Hypoxanthine osf oribosyltransferase

Ксантиноксидаза Xanthine oxidase

Аденозиндезаміназа Adenosine deaminase

Гуаніндезаміназа Guanindeaminase

Аденінфосфорибозилтрансфераза Adenine phosphoribosyltransferase

51 / 200
У 8-місячної дитини спостерігаються блювання та діарея після прийому фруктових соків. Навантаження фруктозою призвело до гіпоглікемії. Спадкова недостатність якого ферменту є причиною стану дитини? An 8-month-old child has vomiting and diarrhea after consuming fruit juices. The fructose load has led to hypoglycemia. An inherited deficiency of which enzyme is the cause of the child's condition?

Гексокіназа Hexokinase

Фруктозо-1-фосфатальдолаза Fructose-1-phosphate aldolase

Фосфофруктокіназа Phosphofructokinase

Фруктокіназа Fructokinase

Фруктозо-1,6-дифосфатаза Fructose-1,6-diphosphatase

52 / 200
При лікуванні пародонтиту використовують препарати кальцію та гормон, що має здатність стимулювати мінералізацію зубів та гальмувати резорбцію кісткової тканини, а саме: In the treatment of periodontitis, calcium preparations and a hormone that has the ability to stimulate the mineralization of teeth and inhibit bone tissue resorption are used, namely:

Альдостерон Aldosterone

Адреналін Adrenaline

Тироксин Thyroxine

Кальцитонін Calcitonin

Паратгормон Parathyroid hormone

53 / 200
Катіонні глікопротеїни є основними компонентами слини привушних залоз. Які амінокислоти обумовлюють їх позитивний заряд? Cationic glycoproteins are the main components of parotid gland saliva. What amino acids determine their positive charge?

Лізин, аргінін, гістидин Lysine, arginine, histidine

Цистеїн, гліцин, пролін Cysteine, glycine, proline

Глутамат, валін, лейцин Glutamate, valine, leucine

Аспартат, аргінін, глутамат Aspartate, arginine, glutamate

Аспартат, глутамат, гліцин Aspartate, glutamate, glycine

54 / 200
З віком знижується секреторна активність привушних слинних залоз. Активність якого ферменту слини буде різко зменшуватись? With age, the secretory activity of the parotid salivary glands decreases. The activity of which salivary enzyme will decrease sharply?

Амілаза Amylase

Мальтаза Maltase

Фосфатаза Phosphatase

Гексокіназа Hexokinase

Лізоцим Lysozyme

55 / 200
Для визначення антитоксичної функції печінки хворому призначено бен-зоат натрію, який в печінці перетворюється в гіпурову кислоту. Яка сполука використовується для цього процесу? To determine the antitoxic function of the liver, the patient is prescribed sodium benzoate, which is converted into hippuric acid in the liver. What compound is used for this process?

Гліцин Glycine

ФАФС FAFS

Цистеїн Cysteine

Метіонін Methionine

УДФ-глюкуронова кислота UDF-glucuronic acid

56 / 200
При спадковій оратацидурії виділення оротової кислоти в багато разів перевищує норму. Синтез яких речовин буде порушений при цій патології? With hereditary orataciduria, the excretion of orotic acid is many times higher than the norm. The synthesis of what substances will be disturbed in this pathology?

Біогенні аміни Biogenic amines

Піримідинові нуклеотиди Pyrimidine nucleotides

Сечовина Urea

Сечова кислота Uric acid

Пуринові нуклеотиди Purine nucleotides

57 / 200
Чоловік на вулиці підняв двома руками електричний дріт, що був під високою напругою. Наступила миттєва смерть від: A man on the street picked up an electric wire with both hands that was under high voltage. Instantaneous death occurred from:

Крововтрати Blood loss

Крововиливу у головний мозок Hemorrhage in the brain

Опіків Burns

Фібриляції серця Heart fibrillation

Зупинки дихання Respiratory arrests

58 / 200
У ліквідатора наслідків аварії на АЕС, що отримав дозу опромінення 5 Гр, через тиждень у крові визначений агранулоцитоз. Який патогенетичний механізм є провідним у його виникненні? The liquidator of the consequences of the accident at the nuclear power plant, who received a radiation dose of 5 Gy, a week later, agranulocytosis was detected in his blood. What pathogenetic mechanism is the leading cause of its occurrence?

Збільшений перехід гранулоцитів у тканини Increased migration of granulocytes into tissues

Порушення виходу зрілих лейкоцитів з кісткового мозку Disturbance of the release of mature leukocytes from the bone marrow

Збільшення руйнування лейкоцитів Increased destruction of leukocytes

Пригнічення лейкопоезу Suppression of leukopoiesis

Розвиток аутоімунного процесу Development of autoimmune process

59 / 200
У хворого, якому було екстирповано зуб з приводу гострого гнійного періоститу, спостерігалася тривала кровотеча з комірки, яку не можна було спинити звичайними методами. У крові: ер.- 2,9 ■ 1012/л, НЬ- 90 г/л; КП- 0,9; тромб.- 60 ■ 109/л; лейк.- 52 ■ 109/л.; базофіли - 0, еозинофіли - 1%, моноцити - 0, нейтрофіли: юні - 0, паличкоядерні -2%, сегментоядерні -18%, лімфоцити -8%, моноцити -1%, мiєлобласти - 70%. Яке захворювання крові наявне у цього хворого? A patient who had a tooth extracted due to acute purulent periostitis had prolonged bleeding from a cell that could not be stopped by conventional methods. In the blood: er.- 2.9 ■ 1012/l, Hb- 90 g/l, KP- 0.9, thromb.- 60 ■ 109/l, leuk.- 52 ■ 109/l., basophils - 0, eosinophils - 1%, monocytes - 0, neutrophils: young - 0, rod-2%, segmentonuclear -18%, lymphocytes -8%, monocytes -1%, myeloblasts - 70%. What blood disease does this patient have?

Хронічний мієлоїдний лейкоз Chronic myeloid leukemia

Еритромієлоз Erythromyelosis

Недиференційований лейкоз Undifferentiated leukemia

Промієлоцитарний лейкоз Promyelocytic leukemia

Гострий мієлоїдний лейкоз Acute myeloid leukemia

60 / 200
Жінка із токсикозом вагітності страждає на гіперсалівацію, що призводить до втрати 3-4 літрів слини щоденно. Яке порушення водно-сольового обміну виникає при цьому? A woman with toxicosis of pregnancy suffers from hypersalivation, which leads to the loss of 3-4 liters of saliva daily. What kind of violation of water-salt metabolism occurs in this case?

Гіпонатріємія Hyponatremia

Гіпогідратація гіперосмолярна Hyporosmolar hypohydration

Гіпогідратація ізоосмолярна Hypohydration isosmolar

Гіпогідратація гіпоосмолярна Hypohydration hypoosmolar

Гіпокаліємія Hypokalemia

61 / 200
Хвора 23-х років скаржиться на виражену слабкість, сонливість, потемніння в очах, запаморочення, спотворення смаку. В анамнезі менорагії. Об’єктивно: блідість шкірних покривів, тріщини в кутах рота, розшаровані нігті, збільшення ЧД і ЧСС. У крові: ер.-2,8 • 1012/л, Hb- 70 г/л, КП- 0,75. Яка гіпоксія, найбільш імовірно, призвела до розвитку виявлених симптомів у хворої? A 23-year-old patient complains of pronounced weakness, drowsiness, darkening of the eyes, dizziness, distortion of taste. She has a history of menorrhagia. Objectively: pale skin, cracks in the corners of the mouth, flaky nails, increased BH and heart rate. In the blood: ER-2.8 • 1012/l, Hb- 70 g/l, CP- 0.75. Which hypoxia most likely led to the development of the identified the patient's symptoms?

Циркуляторна Circulator

Тканинна Fabric

Субстратна Substrate

Респіраторна Respiratory

Гемічна Chemical

62 / 200
У вагітної жінки розвинувся токсикоз з важкими повторними блюваннями упродовж доби. До кінця доби почали проявлятися тетанічні судоми і зневоднення організму. Який зсув кислотно-лужної рівноваги викликав зазначені зміни? A pregnant woman developed toxicosis with severe repeated vomiting throughout the day. By the end of the day, tetanic convulsions and dehydration of the body began to appear. What shift in the acid-base balance caused these changes?

Негазовий видільний ацидоз Nongaseous excretory acidosis

Газовий ацидоз Gas acidosis

Негазовий видільний алкалоз Nongaseous excretory alkalosis

Негазовий метаболічний ацидоз Nongaseous metabolic acidosis

Газовий алкалоз Gas alkalosis

63 / 200
В аналізі крові хворого: ер.- 3,0 • 1012/л; Hb- 90 г/л; ретикулоцити - 0,5%. В мазку: пойкілоцити, гіпохромні еритроцити. Залізо сироватки крові - 80 мкмоль/л. Для якої патології це характерно? In the patient's blood analysis: ER - 3.0 • 1012/l; Hb - 90 g/l; reticulocytes - 0.5%. In the smear: poikilocytes, hypochromic erythrocytes. Blood serum iron - 80 μmol/l. What pathology is this typical for?

B12-дефіцитна анемія B12-deficiency anemia

Хвороба Мінковського-Шофара Minkowski-Shofar disease

Залізорефрактерна анемія Iron refractory anemia

Серпоподібноклітинна анемія Sickle cell anemia

Залізодефіцитна анемія Iron deficiency anemia

64 / 200
Хворого доставили до клініки у коматозному стані. В анамнезі цукровий діабет II типу впродовж 5-ти років. Об’єктивно: дихання шумне, глибоке, у видихуваному повітрі чути запах ацетону. Вміст глюкози у крові 15,2 ммоль/л, кетонових тіл - 100 мкмоль/л. Для якого ускладнення цукрового діабету характерні такі клінічні прояви? The patient was brought to the clinic in a comatose state. He has a history of type II diabetes for 5 years. Objectively: breathing is noisy, deep, and can be heard in exhaled air the smell of acetone. The content of glucose in the blood is 15.2 mmol/l, ketone bodies - 100 μmol/l. What complications of diabetes are characterized by such clinical manifestations?

Гіперглікемічна кома Hyperglycemic coma

Кетоацидотична кома Ketoacidotic coma

Печінкова кома Hepatic coma

Гіпоглікемічна кома Hypoglycemic coma

Гіперосмолярна кома Hyperosmolar coma

65 / 200
При мікроскопічному дослідження пунктату з осередку запалення у хворого із абсцесом шкіри знайдено велику кількість різних клітин крові. Які з цих клітин першими надходять із судин до тканин при запаленні? During a microscopic examination of a punctate from a focus of inflammation in a patient with a skin abscess, a large number of different blood cells were found. Which of these cells are the first to arrive from vessels to tissues during inflammation?

Моноцити Monocytes

Лімфоцити Lymphocytes

Базофіли Basophils

Нейтрофіли Neutrophils

Еозинофіли Eosinophils

66 / 200
Хвора 18-ти років скаржиться на загальну слабкість, швидку втомлюваність, пригнічений настрій. Має астенічний тип конституції. Ps- 68/хв., АТ-90/60 мм рт.ст. Встановлена первинна нейроциркуляторна артеріальна гіпотензія. Що є провідним фактором зниження артеріального тиску у хворої? An 18-year-old patient complains of general weakness, rapid fatigue, depressed mood. She has an asthenic constitution. Ps- 68/min., BP-90/60 mm Hg. Primary neurocirculatory arterial hypotension is established. What is the leading factor in lowering the patient's blood pressure?

Зниження тонусу резистивних судин Decreased tone of resistive vessels

Депонування крові в венах великого кола кровообігу Blood deposition in the veins of the great circulatory circle

Зменшення хвилинного об’єму крові Decrease in minute blood volume

Зменшення серцевого викиду Decreasing cardiac output

Гіповолемія Hypovolemia

67 / 200
Обстежуючи ротову порожнину хворого, стоматолог звернув увагу на наявність запальнодистрофічного процесу в слизовій оболонці (гунтеров-ський глосит, атрофічний стоматит). Аналіз крові виявив гіперхромну анемію. Який фактор є причиною цього захворювання? While examining the patient's oral cavity, the dentist drew attention to the presence of an inflammatory dystrophic process in the mucous membrane (Gunter's glossitis, atrophic stomatitis). Blood analysis revealed hyperchromic anemia. What factor is the cause of this disease?

Гіповітаміноз B1 Hypovitaminosis B1

Гіповітаміноз B12 Hypovitaminosis B12

Підвищення кислотності шлункового соку Increased acidity of gastric juice

Гіповітаміноз B6 Hypovitaminosis B6

Гіповітаміноз A Hypovitaminosis A

68 / 200
У хворого діагностовано хронічний гломерулонефрит. Внаслідок значних склеротичних змін маса функціонуючих нефронів зменшилася до 10%. Яке з перерахованих нижче порушень лежить в основі розвитку в хворого уремічного синдрому? The patient was diagnosed with chronic glomerulonephritis. As a result of significant sclerotic changes, the mass of functioning nephrons decreased to 10%. Which of the following disorders is the basis for the development of uremic syndrome in the patient?

Артеріальна гіпертензія Hypertension

Ниркова остеодистрофія Renal osteodystrophy

Порушення водного гомеостазу Disruption of water homeostasis

Азотемія Azotemia

Порушення осмотичного гомеостазу Disruption of osmotic homeostasis

69 / 200
У хворої жінки 29-ти років місяце-подібне обличчя, ожиріння верхньої частини тулуба, стриї на передній черевній стінці, гірсутизм, у сечі підвищено рівень 17- оксикетостероїдів. Вищезазначені прояви характерні для захворювання: A 29-year-old female patient has a moon-like face, obesity of the upper part of the body, striae on the front abdominal wall, hirsutism, increased level of 17-oxyketosteroids in urine. The above-mentioned manifestations are characteristic of the disease:

Синдром Кона Conn's syndrome

Феохромоцитома

Синдром Іценко-Кушінга Itsenko-Cushing syndrome

Первинний гіпоальдостеронізм Primary hypoaldosteronism

Вторинний гіперальдостеронізм Secondary hyperaldosteronism

70 / 200
На розтині трупа жінки 52-х років, яка тривалий час хворіла на хронічний гломерулонефрит, виявлено: значно зменшені, щільні, дрібнозернисті нирки, фібринозне запалення серозних і слизових оболонок, дистрофічні зміни паренхіматозних органів, набряк головного мозку. Яким ускладненням зумовлені описані зміни серозних оболонок і внутрішніх органів? The autopsy of a 52-year-old woman who suffered from chronic glomerulonephritis for a long time revealed: significantly reduced, dense, fine-grained kidneys, fibrinous inflammation of the serous and mucous membranes , dystrophic changes in parenchymal organs, cerebral edema. What complication is caused by the described changes in the serous membranes and internal organs?

ДВЗ -синдром DVZ syndrome

Тромбоцитопенія Thrombocytopenia

Анемія Anemia

Уремія Uremia

Сепсис Sepsis

71 / 200
Робітник хімічної промисловості звернувся до лікаря зі скаргою на стирання емалі. Об’єктивно: розповсюджене руйнування коронок зубів з утворенням замісного дентину. Який діагноз найбільш вірогідний? A chemical industry worker went to the doctor with a complaint about enamel wear. Objectively: widespread destruction of tooth crowns with the formation of replacement dentine. What diagnosis is most likely?

Некроз твердих тканин зубів Necrosis of hard tissues of teeth

Флюороз Fluorosis

Клиноподібні дефекти Wedge defects

Ерозія зубів Erosion of teeth

Середній карієс Average caries

72 / 200
Жінка 35-ти років звернулася до лікаря зі скаргами на наявність пухлинного утворення на слизовій оболонці ясни у ділянці верхніх різців. Під час огляду лікар на слизовій оболонці ясни виявив вузлик 1 см у діаметрі, м’якої консистенції, рожевого кольору, що кровить під час дотику. Про який процес йдеться у даному випадку? A 35-year-old woman turned to the doctor with complaints about the presence of a tumor on the mucous membrane of the gums in the area of the upper incisors. During the examination, the doctor found a nodule on the mucous membrane of the gums 1 cm in diameter, soft consistency, pink in color, bleeding when touched. What process is involved in this case?

Фіброзний епулід Fibrous epulid

Пухлина Tumor

Гігантоклітинний епулід Giant cell epulid

Хронічний гінгівіт Chronic gingivitis

Ангіоматозний епулід Angiomatous epulid

73 / 200
При гістологічному дослідженні стінки бронха і прилеглих ділянок легені виявлені пласти і тяжі атипового плоского епітелію. У клітинах - помірно виражені ознаки атипізму: поліморфізм, гіперхромія ядер, мітози. У центрі комплексів концентричні утворення рожевого кольору. Який найбільш імовірний діагноз? During histological examination of the wall of the bronchus and adjacent areas of the lung, layers and strands of atypical squamous epithelium were found. In the cells, there are moderately expressed signs of atypism: polymorphism, hyperchromia of nuclei, mitoses. In concentric formations of pink color in the center of the complexes. What is the most likely diagnosis?

Плоскоклітинний рак без ороговіння Squamous cell carcinoma without keratinization

Недиференційований рак Undifferentiated cancer

Скірозний рак Thorough cancer

Аденокарцинома Adenocarcinoma

Плоскоклітинний рак з ороговінням Squamous cell carcinoma with keratinization

74 / 200
При гістологічному дослідженні стінки кісти, що локалізується у ділянці верхньої щелепи, встановлено, що стінка кісти зсередини вистелена багатошаровим плоским епітелієм з підлеглою грануляційною тканиною з лім-фолейкоцитарною інфільтрацією. Зовнішній шар представлений пухкою волокнистою сполучною тканиною, оточеною рубцевою фіброзною тканиною. Ці дані є підставою для встановлення такого діагнозу: During histological examination of the wall of the cyst localized in the area of the upper jaw, it was established that the wall of the cyst is lined from the inside with multilayered flat epithelium with underlying granulation tissue with lympho-leukocyte infiltration. The outer layer is represented by loose fibrous connective tissue, surrounded by scar tissue. These data are the basis for establishing the following diagnosis:

Кератокіста Keratocyst

Амелобластома Ameloblastoma

Кістогранульома Cystogranuloma

Епітеліальна гранульома Epithelial granuloma

Проста гранульома Simple granuloma

75 / 200
При обстеженні у хворого на твердому піднебінні виявлена пухлина у вигляді невеликого щільного вузла сірого кольору без чітких меж. Після видалення пухлина досліджена гістологічно. Побудована з дрібних, кубічної форми клітин з гіперхромним ядром, які формують альвеоли, трабекули, солідні та крі6розні структури. Ріст пухлини - інвазивний. Назвіть пухлину: During the examination, a tumor was found on the patient's hard palate in the form of a small dense gray nodule without clear boundaries. After removal, the tumor was examined histologically. It is made of small, cubic-shaped cells with a hyperchromic nucleus, which form alveoli, trabeculae, solid and crystalline structures. Tumor growth is invasive. Name the tumor:

Мономорфна аденома Monomorphic adenoma

Злоякісна плеоморфна аденома Malignant pleomorphic adenoma

Аденокістозна карцинома Adenocystic carcinoma

Мукоепідермоїдний рак Mucoepidermoid cancer

Аденолімфома Adenolymphoma

76 / 200
У хворої, що страждала на вторинний сифіліс, з’явилися вогнища депігментації шкіри у верхніх відділах спини. Назвіть патологічний процес у ттткі-рі: A patient suffering from secondary syphilis developed foci of skin depigmentation in the upper parts of the back. Name the pathological process in tttki-ri:

Дисплазія Dysplasia

Лейкоплакія Leukoplakia

Паракератоз Parakeratosis

Лейкодерма Leukoderma

Метаплазія Metaplasia

77 / 200
При гістологічному дослідженні збільшеного шийного лімфатичного вузла відмічено, що загальний малюнок вузла нечіткий за рахунок розростання атипових гістіоцитарних клітин, з наявністю гігантських клітин Б ерезовського-Штернберга, маються ділянки некрозу, склерозу, гіалінозу. Для якого захворювання характерні виявлені морфологічні зміни в лімфатичному вузлі? During the histological examination of an enlarged cervical lymph node, it was noted that the general pattern of the node is unclear due to the growth of atypical histiocytic cells, with the presence of giant B cells of Erezovsky-Sternberg, there are areas of necrosis , sclerosis, hyalinosis. What disease is characterized by the detected morphological changes in the lymph node?

Туберкульоз Tuberculosis

Лімфогранулематоз Lymphogranulomatosis

Хронічний мієлолейкоз Chronic myelogenous leukemia

Грибоподібний мікоз Mycosis fungoides

Гострий мієлолейкоз Acute myelogenous leukemia

78 / 200
На розтині тіла померлого чоловіка 70-ти років серце зменшене у розмірах, жирова клітковина під епікардом відсутня, міокард щільний, бурого кольору. При мікроскопії - кардіоміоцити зменшені у розмірах, в саркоплазмі багато гранул бурого пігменту ліпофусцину. Встановіть характер патологічного процесу в міокарді: At the autopsy of a dead man in his 70s, the heart is reduced in size, fatty tissue under the epicardium is absent, the myocardium is dense, brown in color. Under microscopy, cardiomyocytes are reduced in size , there are many granules of brown pigment lipofuscin in the sarcoplasm. Establish the nature of the pathological process in the myocardium:

Бура атрофія Brown atrophy

Амілоїдоз Amyloidosis

Гіпертрофія Hypertrophy

Жирова дистрофія Fat dystrophy

Гемохроматоз Hemochromatosis

79 / 200
У чоловіка 70-ти років, який помер від серцевої недостатності, під час розтину тіла виявлено деформовані, звужені коронарні артерії. На розрізі внутрішня поверхня артерій горбиста, стінка білувата, ламка, кам’янистої щільності. Про яку стадію атеросклерозу йдеться? In a 70-year-old man who died of heart failure, during the autopsy, deformed, narrowed coronary arteries were found. On the section, the inner surface of the arteries is bumpy, the wall is whitish , brittle, stony density. What stage of atherosclerosis are we talking about?

Виразкування Ultration

Атероматоз Atheromatosis

Атерокальциноз Atherocalcinosis

Ліпоїдоз Lipoidosis

Ліпосклероз Liposclerosis

80 / 200
В сироватці крові новонародженого знайдено антитіла до вірусу кору. Про наявність якого імунітету це може свідчити? Antibodies to the measles virus were found in the blood serum of a newborn. What kind of immunity does this indicate?

Штучний активний Artificial Active

Природний пасивний Natural Passive

Спадковий Hereditary

Штучний пасивний Artificial Passive

Природний активний Natural active

81 / 200
До стоматолога звернулась пацієнтка (доярка) із висипанням на слизовій оболонці порожнини рота у вигляді афт. Лікар виявив висипання на шкірі довкола нігтьових пластинок на руках. Який збудник спричинив дану хворобу? A patient (a milkmaid) came to the dentist with a rash on the mucous membrane of the oral cavity in the form of an aphthae. The doctor found a rash on the skin around the nail plates on the hands. What pathogen caused the disease disease?

Вірус ящуру FMD virus

Вірус везикулярного стоматиту Vesicular stomatitis virus

Цитомегаловірус Cytomegalovirus

Герпесвірус Herpesvirus

Вірус Коксакі В Coxsackie B virus

82 / 200
У чоловіка, мешканця сільської місцевості, в шийно-щелепній ділянці виявлено твердий флегмоноподі-бний інфільтрат, шкіра навколо синьо-багряного кольору. В центрі інфільтрат некротизований, з виразки виділяється гній з неприємним запахом. Для підтвердження діагнозу актиномікоз шийно-щелепної ділянки здійснено мікроскопічне дослідження гною. Що повинен виявити бактеріолог для підтвердження діагнозу? In a man, a resident of a rural area, a hard phlegmon-like infiltrate was detected in the neck and jaw area, the skin around it is blue-purple in color. In the center, the infiltrate is necrotic, from an ulcer pus with an unpleasant smell is released. To confirm the diagnosis of actinomycosis of the cervical-maxillary area, a microscopic examination of the pus was performed. What should the bacteriologist find to confirm the diagnosis?

Кислотостійкі палички Acid-resistant sticks

Грамнегативні диплобактерії Gram-negative diplobacteria

Грамнегативні диплококи Gram-negative diplococci

Грампозитивні стрептококи Gram-positive streptococci

Друзи Friends

83 / 200
Бактеріолог при дослідженні крові та слизу із носоглотки дотримувався певних заходів щодо збереження збудників у матеріалі. При бактеріоскопі-чному дослідженні встановлено наявність грамнегативних коків, які нагадують кавові зерна і розташовані парами, або тетрадами. Назвіть збудника, який був ізольований бактеріологом: During the examination of blood and mucus from the nasopharynx, the bacteriologist observed certain measures to preserve pathogens in the material. During the bacterioscopic examination, the presence of gram-negative cocci, which resemble coffee beans and are located in pairs or tetrads. Name the pathogen that was isolated by the bacteriologist:

Acinetobacter calcoaceticus Acinetobacter calcoaceticus

Neisseria gonorrhoeae Neisseria gonorrhoeae

Neisseria meningitidis Neisseria meningitidis

Moraxella lacunata Moraxella lacunata

Staphilococcus aureus Staphilococcus aureus

84 / 200
Хворому з підозрою на черевний тиф лікар-інфекціоніст призначив бактеріологічне дослідження крові. Доцільність цього призначення пояснюється тим, що на першому тижні захворювання тифо-паратифами спостерігається: An infectious disease doctor ordered a bacteriological blood test for a patient with suspected typhoid fever. The expediency of this appointment is explained by the fact that in the first week of the typhoid-paratyphoid illness:

Септицемія Septicemia

Вірусемія Virusemia

Токсинемія Toxinemia

Септикопіємія Septicopyemia

Бактеріємія Bacteremia

85 / 200
Від хворого з підозрою на черевний тиф виділено чисту культуру збудника, яку ідентифіковано за морфологічними, культуральними та біохімічними властивостями як сальмонела тифу. Яке дослідження слід застосувати для остаточної ідентифікації збудника? A pure culture of the pathogen was isolated from a patient with suspected typhoid fever, which was identified by morphological, cultural and biochemical properties as Salmonella typhus. What research should be used to definitively identify the pathogen ?

Антибіотикограма Antibiotic program

Сероідентифікація Seroidentification

Серодіагностика Serodiagnosis

Фаготипування Phagotyping

Алергодіагностика Allergo diagnosis

86 / 200
Хворому з афтами слизової оболонки призначили препарат, діючим агентом котрого є галоген, а також поверхнево активна речовина, що має дезін-фікуючу, дезодоруючу дію. Застосовується для дезінфекції неметалевого інструментарію, рук, предметів догляду за інфекційними хворими. Як антисептик використовується для лікування інфікованих ран, слизової оболонки ротової порожнини, патологічних зубоя-сеневих кишень, дезінфекції кореневих каналів. Визначте препарат: A patient with aphthae of the mucous membrane was prescribed a drug, the active agent of which is a halogen, as well as a surface-active substance that has a disinfecting, deodorizing effect. It is used to disinfect non-metallic instruments, hands, items for the care of infectious patients. As an antiseptic, it is used for the treatment of infected wounds, the mucous membrane of the oral cavity, pathological tooth and sinus pockets, disinfection of root canals. Define the drug:

Калію перманганат Potassium permanganate

Діамантовий зелений Diamond Green

Хлоргексидину біглюконат Chlorhexidine bigluconate

Кислота борна Boric acid

Перекис водню Hydrogen peroxide

87 / 200
При введенні хворому з метою проведення провідникової анестезії засобу, який використовується в хірургічній стоматології, виникли симптоми отруєння: збудження ЦНС з наступним паралічем, гостра серцево-судинна недостатність (колапс), в патогенезі якого має значення сенсибілізація до даного препарату. Виникли також алергі- чні реакції (свербіж, набряклість, еритема). Визначте препарат: When a drug used in surgical dentistry was administered to the patient for the purpose of conduction anesthesia, symptoms of poisoning occurred: excitation of the central nervous system followed by paralysis, acute cardiovascular insufficiency (collapse ), in the pathogenesis of which sensitization to this drug is important. Allergic reactions (itching, swelling, erythema) also occurred. Define the drug:

Тіопентал-натрію Thiopental sodium

Тубокурарину хлорид Tubocurarine chloride

Лідокаїн Lidocaine

Дитилін Ditylin

Ардуан Arduan

88 / 200
В комплексному лікуванні гінгівіту хворому призначили препарат, який за хімічною будовою відноситься до похідних піримідину, стимулює лейкопоез, прискорює загоєння ран, підсилює ріст та розмноження клітин (процеси проліферації), виявляє протизапальну дію. Застосовується при лейкопеніях різного генезу, в стоматологічній практиці при запальних захворюваннях слизової оболонки ротової порожнини. Визначте препарат: In the complex treatment of gingivitis, the patient was prescribed a drug that chemically belongs to pyrimidine derivatives, stimulates leukopoiesis, accelerates wound healing, enhances cell growth and reproduction (proliferation processes) , has an anti-inflammatory effect. It is used for leukopenia of various genesis, in dental practice for inflammatory diseases of the mucous membrane of the oral cavity. Define the drug:

Коамід Coamide

Метотрексат Methotrexate

Меркаптопурин Mercaptopurine

Метилурацил Methyluracil

Ціанокобаламін Cyanocobalamin

89 / 200
У хворої з артеріальною гіпертензією розвинувся напад бронхіальної астми. Який з нижче перерахованих бронхолітичних засобів може спровокувати гіпертонічний криз? A patient with arterial hypertension developed an attack of bronchial asthma. Which of the following bronchodilator agents can provoke a hypertensive crisis?

Ьадрин Hadrin

Сальбутамол Salbutamol

Ефедрину гідрохлорид Ephedrine hydrochloride

Еуфілін Euphilin

Беротек Berotek

90 / 200
Для корекції артеріального тиску при колаптоїдному стані хворому було введено мезатон. Який механізм дії даного препарату? Mezaton was administered to the patient to correct blood pressure in a colaptoid condition. What is the mechanism of action of this drug?

Стимулює а- в-адренорецептори Stimulates α-β-adrenoceptors

Стимулює а-адренорецептори Stimulates α-adrenoceptors

Блокує а-адренорецептори Blocks α-adrenoceptors

Стимулює в-адренорецептори Stimulates v-adrenoceptors

Блокує в-адренорецептори Blocks v-adrenoceptors

91 / 200
Після повторного оперативного втручання з використанням того ж самого загального анестетика у хворого виникло гостре ураження печінки (гепатит). Який загальний анестетик найбільш імовірно може викликати дану патологію? After repeated surgery using the same general anesthetic, the patient developed acute liver damage (hepatitis). Which general anesthetic is most likely to cause this pathology?

Азоту закис Nitric oxide

Пропанідид Propanidide

Фторотан Fluorotan

Тіопентал натрію Sodium thiopental

Ефір Ether

92 / 200
На прийомі у стоматолога у хворого виник гострий біль за грудниною, який іррадіює в ліву руку і плече; відчуття нестачі повітря, страх. Яка з запропонованих речовин найбільш адекватна в цьому випадку? During an appointment with a dentist, the patient developed a sharp pain behind the sternum that radiates to the left arm and shoulder; a feeling of lack of air, fear. Which of the proposed substances is the most adequate in in this case?

Діазепам Diazepam

Нітросорбід Nitrosorbide

Нітрогліцерин Nitroglycerin

Парацетамол Paracetamol

Анальгін Analgin

93 / 200
Під час оперативного втручання анестезіолог для керованої гіпотонії застосував гангліоблокуючий засіб. Який препарат було призначено хворому в даному випадку? During the surgical intervention, the anesthesiologist used a ganglion-blocking drug for controlled hypotension. What drug was prescribed to the patient in this case?

Бензогексоній Benzohexonium

Гігроній Hygronium

Пентамін Pentamine

Пахікарпін Pachycarpine

Пірілен Pyrylene

94 / 200
До поліклініки звернувся хворий зі скаргами на біль за грудниною, зади-тттку і серцебиття. Після обстеження лікар діагностував у хворого ІХС і призначив верапаміл. Який механізм дії даного препарату? A patient came to the polyclinic with complaints of chest pain, back pain, and heart palpitations. After the examination, the doctor diagnosed the patient with coronary artery disease and prescribed verapamil. What is the mechanism of action of this drug ?

Блокує кальцієві канали Blocks calcium channels

Блокує калієві канали Blocks potassium channels

Блокує в-адренорецептори Blocks v-adrenoceptors

Блокує натрієві канали Blocks sodium channels

Блокує а-адренорецептори Blocks α-adrenoceptors

95 / 200
Хворій потрібно зробити пункцію прямокишково-маткової заглибини (Дугласового простору). Через яку частину склепіння піхви її здійснюють? The patient needs to have a rectal-uterine puncture (Douglas space). Through which part of the vaginal vault is it performed?

Бічна права Side right

Будь-яка Any

Задня Back

Передня Front

Бічна ліва Side Left

96 / 200
Для попередження сезонного підйому захворюваності на грип у лікувальних закладах міста санепідстанція зобов’язала провести імунізацію медпрацівників. Яким із перелічених препаратів слід проводити імунізацію? In order to prevent a seasonal increase in the incidence of influenza in the city's medical institutions, the sanitary substation obliged to carry out immunization of medical workers. Which of the listed drugs should be used for immunization?

Гамма-глобулін Gamma Globulin

Амантадин Amantadine

Ремантадин Remantadine

Інтерферон Interferon

Субодинична вакцина Subunit vaccine

97 / 200
У хворого на шкірі живота, грудної клітки з’явився висип у вигляді розеол і петехій, дрібноточковий кон’юн-ктивальний висип. Смерть настала при явищах ураження мозку. При мікроскопічному дослідженні секційного матеріалу у ЦНС (довгастий мозок, міст), у шкірі і нирках, міокарді виявлений деструктивно-проліферативний ендо-тромбоваскуліт. Про яке захворювання слід думати? The patient developed a rash in the form of roseolae and petechiae on the skin of the abdomen and chest, small point conjunctival rash. Death occurred due to brain damage. A microscopic examination of the section material in the central nervous system (medulla, bridge), in the skin and kidneys, in the myocardium revealed destructive-proliferative endo-thrombovasculitis. What disease should we think about?

Вузликовий періартеріїт Nodular periarteritis

Висипний тиф Typhoid

Бруцельоз Brucellosis

Сепсис Sepsis

Системний червоний вівчак System red sheep

98 / 200
В хірургічне відділення доставлено чоловіка 35-ти років з гнійною раною на шиї попереду трахеї (в ділянці пере-двісцерального простору). Куди може розповсюдитись інфекція, якщо хворому терміново не зробити операцію? A 35-year-old man was brought to the surgical department with a purulent wound on the neck in front of the trachea (in the area of the pre-visceral space). Where can the infection spread if the patient urgently not do the operation?

В ретровісцеральний простір Into retrovisceral space

В надгрудинний міжапоневротичний простір In the suprasternal interaponeurotic space

В грудну порожнину - в середнє середостіння In the chest cavity - in the middle mediastinum

В грудну порожнину - переднє середостіння Into the chest cavity - anterior mediastinum

В грудну порожнину - в заднє середостіння In the chest cavity - in the posterior mediastinum

99 / 200
У деяких клінічно здорових людей в умовах високогір’я виявляються ознаки анемії. У крові в них виявляють серпоподібні еритроцити. Генотип цих людей: Some clinically healthy people in the highlands show signs of anemia. Sickle-shaped erythrocytes are found in their blood. Genotype of these people:

Аа Ah

ХсХс XxXx

ХСХс ХСХс

АА AA

аа aa

100 / 200
У пацієнта з хронічною серцевою недостатністю виявлене збільшення в’язкості крові. При капіляроскопії виявлено пошкодження стінок судин мікроциркуляторного русла. Яке з перелічених порушень можливе у даному випадку? In a patient with chronic heart failure, an increase in blood viscosity was detected. During capillaroscopy, damage to the walls of vessels of the microcirculatory channel was detected. Which of the listed violations is possible in this case?

Тромбоз Thrombosis

Сладж-феномен Sludge phenomenon

Венозна гіпєрємія Venous hyperemia

Артеріальна гіперемія Arterial hyperemia

Емболія Embolism

101 / 200
При обстеженні хворого 35-ти років проведено гістологічне дослідження пунктату червоного кісткового мозку і виявлено значне зменшення кількості мегакаріоцитів. До яких змін периферичної крові це призведе? During the examination of a 35-year-old patient, a histological examination of the red bone marrow punctate was performed and a significant decrease in the number of megakaryocytes was found. What changes in the peripheral blood will this lead to?

Тромбоцитопенія Thrombocytopenia

Лейкопенія Leukopenia

Лейкоцитоз Leukocytosis

Тромбоцитоз Thrombocytosis

Агранулоцитоз Agranulocytosis

102 / 200
Під час проведення трахеотомії у хворого 45-ти років, який потрапив до реанімаційного відділення лікарні з набряком гортані, було випадково перерізано яремну венозну дугу, яка лежить у: During a tracheotomy in a 45-year-old patient who was admitted to the hospital's intensive care unit with laryngeal edema, the jugular venous arch was accidentally cut, which lies in:

Spatium pretracheale Spatium pretracheale

Spatium interscalenum Spatium interscalenum

Spatium antescalenum Spatium antescalenum

Spatium interaponeuroticum suprasternale Spatium interaponeuroticum suprasternale

Spatium retropharyngeale Spatium retropharyngeale

103 / 200
Під час виконання фізичного навантаження людина менш чутлива до болю. Причиною цього є активація в організмі: During physical activity, a person is less sensitive to pain. The reason for this is activation in the body:

Ноцицептивної системи Nociceptive system

Симпато-адреналової системи Sympatho-adrenal system

Функції наднирників Adrenal functions

Функції щитоподібної залози Thyroid functions

Антиноцицептивної системи Antinociceptive system

104 / 200
У жінки з резус-негативною кров’ю А (II) групи народилася дитина з АВ (IV) групою, у якої діагностували гемолітичну хворобу внаслідок резус-конфлікту. Яка група крові можлива у батька дитини? A woman with rhesus-negative blood of group A (II) had a child with group AB (IV), who was diagnosed with hemolytic disease as a result of rhesus conflict. What blood group is possible for the child's father?

IV (АВ), резус-негативна IV (AB), Rh-negative

III (В), резус-позитивна III (B), Rh-positive

I (0), резус-позитивна I (0), Rh positive

II (А), резус-позитивна II (A), Rh-positive

III (В), резус-негативна III (B), Rh-negative

105 / 200
До реанімаційного відділення госпіталізовано хворого з клінікою отруєння нейротропною речовиною. До комплексу лікування включено дихальний аналептик з ноотропною дією. Який препарат призначено? A patient with clinical signs of poisoning with a neurotropic substance was admitted to the intensive care unit. A respiratory analeptic with a nootropic effect was included in the treatment complex. What drug was prescribed?

Камфора Camphor

Етимізол Etimizole

Теофілін Theophylline

Сульфокамфокаїн Sulfocamphocaine

Амітриптилін Amitriptyline

106 / 200
У хворого з’явилися жовтушність шкіри, склер та слизових оболонок. У плазмі крові підвищений рівень загального білірубіну, в калі - стеркобіліну, в сечі - уробіліну. Який вид жовтяниці у хворого? The patient developed yellowing of the skin, sclera, and mucous membranes. The level of total bilirubin in the blood plasma, stercobilin in the feces, and urobilin in the urine is elevated. What type jaundice in the patient?

Холестатична Cholestatic

Гемолітична Hemolytic

Паренхіматозна Parenchymatous

Обтураційна Obstructive

Хвороба Жільбера Gilbert's disease

107 / 200
У хворого внаслідок черепно-мозкової травми з’явилась значна поліурія, що не супроводжувалася глюкозурією. Пошкодження якої структури може призвести до подібних змін? The patient developed significant polyuria as a result of a brain injury, which was not accompanied by glucosuria. Damage to which structure can lead to such changes?

Мозкова речовина наднирників Adrenal brain matter

Передня частка гіпофізу Anterior pituitary lobe

Кора наднирників Adrenal cortex

Задня частка гіпофізу Posterior lobe of the pituitary gland

Підшлункова залоза Pancreas

108 / 200
У дитини 12-ти років низький зріст при непропорційній будові тіла і розумовій відсталості. Недостатня секреція якого гормону може бути причиною таких порушень? A 12-year-old child has short stature with a disproportionate body structure and mental retardation. Insufficient secretion of which hormone can be the cause of such disorders?

Кортизол Cortisol

Соматотропін Somatotropin

Тироксин Tyroxin

Глюкагон Glucagon

Інсулін Insulin

109 / 200
При дослідженні культури тканин злоякісної пухлини виявили поділ клітин, який відбувався без ахроматиново-го апарату шляхом утворення перетяжки ядра, при якому зберігались ядерна оболонка та ядерце. Який тип поділу клітин відбувався у злоякісній пухлині, що вивчалась? During the examination of the tissue culture of a malignant tumor, cell division was found, which occurred without the achromatin apparatus through the formation of a narrowing of the nucleus, in which the nuclear envelope and nucleolus were preserved. What type of division of cells occurred in the malignant tumor under study?

Мітоз Mitosis

Мейоз Meiosis

Ендомітоз Endomitosis

Амітоз Amitosis

Екзомітоз Exomitosis

110 / 200
Хворому проведено лобектомію правої середньої частки легені. Які сегменти легені були уражені? The patient underwent lobectomy of the right middle lobe of the lung. Which lung segments were affected?

Задній і бічний основні Back and side main

Бічний та присередній Lateral and medial

Присередній і передній основні Middle and front primary

Верхівковий, передній Apical, anterior

Верхівково-задній та передній Top-posterior and anterior

111 / 200
У чоловіка 60-ти років, який страждає на хронічну кишкову непрохідність, посилюється гниття білків у товстому кишечнику. Підтвердженням цього процесу є: A 60-year-old man suffering from chronic intestinal obstruction has increased protein decay in the large intestine. Evidence of this process is:

Білірубінурія Bilirubinuria

Гіперурікурія Hyperuricuria

!ндиканурія !ndikanuria

Креатинурія Creatinuria

Глюкозурія Glucosuria

112 / 200
У 19-ти місячної дитини із затримкою розвитку та проявами самоагре-сії вміст сечової кислоти в крові - 1,96 ммоль/л. При якому метаболічному порушенні це спостерігається? In a 19-month-old child with developmental delay and manifestations of self-aggression, the content of uric acid in the blood is 1.96 mmol/l. In which metabolic disorder is this observed ?

Хвороба Іценко-Кушінга Itsenko-Cushing disease

Хвороба Гірке Bitter disease

Синдром набутого імунодефіциту Acquired immunodeficiency syndrome

Синдром Леша-Ніхана Lesch-Nyhan syndrome

Подагра Gout

113 / 200
Хворий у непритомному стані доставлений швидкою допомогою до лікарні. Об’єктивно: рефлекси відсутні, періодично з’являються судоми, дихання нерівномірне. Після лабораторного обстеження було діагностовано печінкову кому. Нагромадження якого метаболіту є суттєвим для появи розладів центральної нервової системи? The patient was taken by ambulance to the hospital in an unconscious state. Objectively: reflexes are absent, convulsions appear periodically, breathing is irregular. After a laboratory examination, hepatic coma was diagnosed . The accumulation of which metabolite is essential for the appearance of disorders of the central nervous system?

Сечовина Urea

Білірубін Bilirubin

Глутамін Glutamine

Гістамін Histamine

Амоніак Ammonia

114 / 200
Під час гістологічного дослідження нирок жінки 25-ти років, що померла під час пологів, в епітелії канальців головних відділів нефрону виявлені конденсація хроматину ядер, розпад їх на глибки та лізис, а також плазморексис та цитоліз. Який патологічний процес виявлений в епітелії канальців нирок? During a histological examination of the kidneys of a 25-year-old woman who died during childbirth, condensation of nuclear chromatin, their disintegration into deep and lysis, as well as plasmorhexis and cytolysis. What pathological process was detected in the epithelium of the kidney tubules?

Некроз Necrosis

Амілоїдоз Amyloidosis

Гіаліноз Hyalinosis

Гідропічна дистрофія Hydropic dystrophy

Жирова дистрофія Fat dystrophy

115 / 200
У хворого на діарею небактеріаль-ного генезу скарги на часту дефекацію. Який з перелічених засобів, механізм протидіарейної дії якого реалізується через опіатні рецептори кишок, доцільно використати? A patient with diarrhea of non-bacterial genesis complains of frequent defecation. Which of the listed remedies, the mechanism of antidiarrheal action of which is implemented through the opiate receptors of the intestines, should be used?

Де-нол De-zero

Но-шпа No-shpa

Лінекс Linux

Атропіну сульфат Atropine sulfate

Імодіум (лоперамід) Imodium (loperamide)

116 / 200
У хворого, що надійшов до інфекційного відділення зі скаргами на судомне скорочення м’язів обличчя, з садна правої нижньої кінцівки були виділені бактерії з термінальним розташуванням спор, що надає їм вигляд 'барабанних паличок'. Яким бактеріям притаманні дані властивості? In a patient who came to the infectious disease department with complaints of convulsive contraction of the facial muscles, bacteria with a terminal location of spores were isolated from the wound of the right lower limb, which gives they look like 'drumsticks'. Which bacteria have these properties?

Bacillus cereus Bacillus cereus

Clostridium botulinum Clostridium botulinum

Clostridium perfringens Clostridium perfringens

Clostridium tetani Clostridium tetani

Bacillus anthracis Bacillus anthracis

117 / 200
Встановлено, що швидкість проведення збудження нервовими волокнами становить 120 м/сек. Зазначені волокна є: It was established that the speed of conduction of excitation by nerve fibers is 120 m/sec. The specified fibers are:

Прегангліонарними парасимпатичними Preganglionic parasympathetic

Постгангліонарними симпатичними Postganglionic sympathetic

Аксонами мотонейронів Axons of motor neurons

Постгангліонарними парасимпатичними Postganglionic parasympathetic

Прегангліонарними симпатичними Preganglionic sympathetic

118 / 200
У дитини фарингіт (запалення слизової оболонки глотки) ускладнився запаленням середнього вуха. Через яке анатомічне утворення сталося розповсюдження запального процесу на середнє вухо? A child's pharyngitis (inflammation of the mucous membrane of the pharynx) was complicated by inflammation of the middle ear. Through which anatomical formation did the inflammation spread to the middle ear?

Півканал слухової труби Hemichannel of the auditory tube

Глотковий отвір слухової труби Pharyngeal opening of the auditory tube

Каналець барабанної струни Tympanic tube

Півканал м’яза-натягувача барабанної перетинки Hemichannel of the tensor tympani muscle

М ’язово-трубний канал Muscular-tubular channel

119 / 200
Під час бійки у чоловіка виникла рефлекторна зупинка серця внаслідок сильного удару у верхню ділянку передньої черевної стінки. На яку структуру серця впливають еферентні нерви, що зумовили його зупинку? During the fight, the man suffered a reflex cardiac arrest as a result of a strong blow to the upper part of the anterior abdominal wall. What structure of the heart is affected by the efferent nerves that caused his arrest?

Робочий мюкард шлуночків Working ventricular myocardium

Провідна система шлуночків серця The conduction system of the ventricles of the heart

Синоатріальний вузол Sinoatrial node

Атріовентрикулярний вузол Atrioventricular node

Робочий міокард передсердь Working atrial myocardium

120 / 200
У хворого спостерігаються геморагії, в крові знижена концентрація протромбіну. Недостатність якого вітаміну призвела до порушення синтезу цього фактору згортання? The patient has hemorrhages, the concentration of prothrombin in the blood is reduced. The lack of which vitamin led to a violation of the synthesis of this coagulation factor?

A A

D D

C C

E E

K K

121 / 200
У хворого, що харчувався винятково полірованим рисом, недостатність тіаміну стала причиною поліневриту. Екскреція якої сполуки з сечею може бути індикатором цього авітамінозу? In a patient who ate exclusively polished rice, thiamine deficiency caused polyneuritis. Excretion of which compound with urine can be an indicator of this vitamin deficiency?

Сечова кислота Uric acid

Піровиноградна кислота Pyruvic acid

Малат Malat

Метил-малонова кислота Methylmalonic acid

Фенілпіруват Phenylpyruvate

122 / 200
Хворому на ревматоїдний артрит для попередження можливого негативного впливу на слизову шлунка призначили препарат із групи нестероїдних протизапальних засобів - селективний інгібітор ЦОГ-2. Вкажіть препарат: A patient with rheumatoid arthritis was prescribed a drug from the group of nonsteroidal anti-inflammatory drugs - a selective COX-2 inhibitor to prevent a possible negative effect on the gastric mucosa. Specify the drug:

Ацетилсаліцилова кислота Acetylsalicylic acid

Целекоксиб Celecoxib

Бутадіон Butadione

Ібупрофен Ibuprofen

Анальгін Analgin

123 / 200
Хворий на стенокардію приймає ацетилсаліцилову кислоту в дозі 100 мг на добу. З якою метою застосовується ацетилсаліцилова кислота у даного хворого? A patient with angina takes acetylsalicylic acid in a dose of 100 mg per day. For what purpose is acetylsalicylic acid used in this patient?

Для пригнічення зсідання крові To inhibit blood clotting

Для зниження вмісту холестерину To lower cholesterol

Для пригнічення агрегації тромбоцитів To inhibit platelet aggregation

Для розширення коронарних судин For expansion of coronary vessels

Для зниження рівня протромбіну To reduce the level of prothrombin

124 / 200
Пацієнт скаржиться на свербіж шкіри, особливо між пальцями рук, у пахвинних западинах, на нижній частині живота. При огляді в цих ділянках шкіри виявлено маленькі пухирці. Під час лабораторної діагностики встановлено, що причиною цього стану є представник членистоногих. Вкажіть назву хвороби, спричинену цим членистоногим: The patient complains of itching of the skin, especially between the fingers, in the groin, on the lower part of the abdomen. During the examination, small blisters were found in these areas of the skin. During the laboratory diagnosis has established that the cause of this condition is an arthropod representative. Enter the name of the disease caused by this arthropod:

Короста Scabies

Дерматотропний лейшманіоз Dermatotropic leishmaniasis

Міаз Miaz

Педикульоз Pediculosis

Демодекоз Demodecosis

125 / 200
У людини внаслідок втрати 1,5 л крові різко зменшився діурез. Посилена секреція якого гормону, перш за все, спричинила зміни діурезу? Due to the loss of 1.5 liters of blood, diuresis decreased dramatically in a person. Increased secretion of which hormone primarily caused changes in diuresis?

Натрійуретичний Natriuretic

Вазопресин Vasopressin

Кортизон Cortisone

Окситоцин Oxytocin

Паратгормон Parathyroid hormone

126 / 200
При обстеженні хворого на атрофічний гастрит виявлено мегалобла-стну анемію. Дефіцит якої речовини є причиною виникнення анемії у цього хворого? During the examination of a patient with atrophic gastritis, megaloblastic anemia was detected. The deficiency of which substance is the cause of anemia in this patient?

Залізо Iron

Гастромукопротеїд Gastromucoprotein

Еритропоетини Erythropoietins

Вітамін Ві Vitamin Vi

Вітамін В6 Vitamin B6

127 / 200
Після відкривання рота відбувається його рефлекторне закривання. З яких рецепторів починається зазначений рефлекс? After opening the mouth, it reflexively closes. From which receptors does the specified reflex begin?

Механорецептори слизової ротової порожнини Mechanoreceptors of the oral mucosa

Смакові рецептори Taste buds

Пропріорецептори м’язів, що піднімають нижню щелепу Proprioreceptors of the muscles that raise the lower jaw

Пропріорецептори м’язів, що опускають нижню щелепу Proprioreceptors of muscles that lower the lower jaw

Рецептори періодонту Periodontal receptors

128 / 200
У дитини 2-х років діагностовано хворобу Гірке, яка проявляється важкою гіпоглікемією. Причиною такого стану є відсутність ферменту глюкозо-6-фосфатази. З порушенням якого процесу пов’язана ця патологія? A 2-year-old child was diagnosed with Gierke's disease, which is manifested by severe hypoglycemia. The cause of this condition is the absence of the glucose-6-phosphatase enzyme. What process is associated with a violation of What is this pathology?

Синтез глікогену Glycogen synthesis

Гліколіз Glycolysis

Мобілізація глікогену Glycogen mobilization

Кетогенез Ketogenesis

Ілюконеогенез Iluconeogenesis

129 / 200
Хворому на хронічний гастрит зроблена внутрішньошлункова рН-метрія, за допомогою якої встановлено змен- шення кислотності шлункового соку. Функція яких клітин знижена? Intragastric pH-metry was performed on a patient with chronic gastritis, with the help of which a decrease in the acidity of gastric juice was established. The function of which cells is reduced?

Головні екзокриноцити Main exocrinocytes

Парiєтальнi екзокриноцити Parietal exocrinocytes

Ендокриноцити Endocrinocytes

Додаткові клітини Additional Cells

Шийкові клітини Cervical cells

130 / 200
Бригада швидкої допомоги була викликана до хворого 48-ми років із приводу інфаркту міокарда. У комплексну терапію було включено анальге-тик, що застосовують для нейролепта-налгезії. Вкажіть найбільш доцільний швидкодіючий препарат: The ambulance was called to a 48-year-old patient due to a myocardial infarction. An analgesic used for neuroleptic analgesia was included in the complex therapy. Specify the most appropriate fast-acting drug:

Фентаніл Fentanyl

Аспірин Aspirin

!ндометацин !ndometacin

Промедол Promedol

Морфіну гідрохлорид Morphine hydrochloride

131 / 200
Відомо, що при цукровому діабеті у хворих частіше зустрічаються запальні процеси, знижена регенерація, уповільнюється загоєння ран. Причиною цього є: It is known that in patients with diabetes, inflammatory processes occur more often, regeneration is reduced, and wound healing slows down. The reason for this is:

Посилення катаболізму Strengthening catabolism

Прискорення глюконеогенезу Acceleration of gluconeogenesis

Зниження ліполізу Decreased lipolysis

Зниження протеосинтезу Reduction of proteosynthesis

Підвищення ліполізу Increasing lipolysis

132 / 200
Жінка звернулася до лікаря зі скаргами на утруднення рухів язика. Обстеження головного мозку за допомогою ЯМР показало, що у хворої є крововилив у нижньому відділі довгастого мозку. Про пошкодження якого ядра довгастого мозку можна думати? The woman went to the doctor complaining of difficulty in moving the tongue. An MRI brain examination showed that the patient had a hemorrhage in the lower part of the medulla oblongata. About the damage to which the nucleus of the medulla can think?

Нижнього слиновидільного ядра Inferior salivary nucleus

Ядра під’язикового нерва Nuclei of the hypoglossal nerve

Поодинокого ядра Single Core

Подвійного ядра Dual Core

Ядра додаткового нерва Supplemental nerve nuclei

133 / 200
При токсичному ушкодженні гепа-тоцитів з порушенням їх білковосинте-зуючої функції у пацієнта різко знизився вміст альбумінів у плазмі крові та онкотичний тиск плазми. Що буде наслідком цих змін? With toxic damage to hepatocytes with a violation of their protein-synthesizing function, the patient's albumin content in the blood plasma and oncotic pressure of the plasma decreased sharply. What will be the consequence of these changes?'

Поява набряків Appearance of edema

Збільшення густини крові Increasing blood density

Зменшення ШОЕ Reduction of ESR

Збільшення об’єму циркулюючої крові Increase in circulating blood volume

Зменшення діурезу Diuresis reduction

134 / 200
У хворого діагностовано грип. Після прийому антипіретиків стан його різко погіршився: свідомість потьмарена, АТ- 80/50 мм рт.ст., Ps- 140/хв, температура тіла різко знизилась до 35,8oC. Яке ускладнення виникло у даного хворого? The patient was diagnosed with influenza. After taking antipyretics, his condition deteriorated sharply: his consciousness was dimmed, blood pressure - 80/50 mm Hg, Ps - 140/min, the body temperature dropped sharply to 35.8oC. What complication did this patient have?

Алкалоз Alkalosis

Гіповолемія Hypovolemia

Колапс Collapse

Гіпертермія Hyperthermia

Ацидоз Acidosis

135 / 200
У хворого лікар виявив накопичення рідини в плевральній порожнині справа над куполом діафрагми. У якому анатомічному утворі накопичилась рідина? In the patient, the doctor discovered fluid accumulation in the pleural cavity on the right above the dome of the diaphragm. In which anatomical structure did the fluid accumulate?

Реброво-діафрагмальний правий синус Cost-phrenic right sinus

- -

Реброво-середостінний синус Cost-mediastinal sinus

Діафрагмально-середостінний синус Diaphragmatic-mediastinal sinus

Реброво-діафрагмальний лівий синус Cost-phrenic left sinus

136 / 200
Пацієнт звернувся до лікаря з приводу того, що він втратив здатність розрізняти смаки на корені язика. Лікар встановив, що це пов’язано з ураженням нерва. Якого саме? A patient consulted a doctor about the fact that he lost the ability to distinguish tastes at the root of the tongue. The doctor determined that this was due to nerve damage. Which one exactly?

Язикоглотковий Lingopharyngeal

Лицьовий Face

Верхньогортанний Upper laryngeal

Трійчастий Triple

Блукаючий Wandering

137 / 200
Введення знеболюючого пацієнту перед екстракцією зуба призвело до розвитку анафілактичного шоку, який супроводжувався розвитком олігурії. Який патогенетичний механізм зумовив зменшення діурезу в даній клінічній ситуації? The administration of an anesthetic to a patient before tooth extraction led to the development of anaphylactic shock, which was accompanied by the development of oliguria. What pathogenetic mechanism caused the decrease in diuresis in this clinical situation?

Зниження гідростатичного тиску в капілярах ниркових тілець Reduction of hydrostatic pressure in the capillaries of the renal corpuscles

Підвищення гідростатичного тиску в капсулі Шумлянського-Боумена Increase in hydrostatic pressure in the Shumlyansky-Bowman capsule

Збільшення онкотичного тиску плазми крові Increase in oncotic pressure of blood plasma

Пошкодження клубочкового фільтру Damage of the glomerular filter

Збільшення секреції вазопресину Increased vasopressin secretion

138 / 200
При втручанні з метою лікування вивиху нижньої щелепи лікар повинен пам’ятати про м’яз, який при скороченні відтягує назовні капсулу і суглобовий диск скроневонижньощелепного суглоба. Який це м’яз? When intervening for the treatment of dislocation of the lower jaw, the doctor must remember the muscle that, when contracted, pulls the capsule and articular disc of the temporomandibular joint outward. What is this m 'I'm from?

M. mylohyoideus M. mylohyoideus

M. pterygoideus medialis M. pterygoideus medialis

M. pterygoideus lateralis M. pterygoideus lateralis

M. masseter M. masseter

M. temporalis M. temporalis

139 / 200
Одним із методів зняття гострого болю при невралгії трійчастого нерва є створення депо анестетика в місці виходу гілок трійчастого нерва. В якій ділянці потрібно ввести анестетик для знечулення першої гілки трійчастого нерва? One of the methods to relieve acute pain in trigeminal neuralgia is to create a depot of anesthetic at the exit point of the branches of the trigeminal nerve. In which area should the anesthetic be injected to anesthetize the first branch of the trigeminal nerve ?

Надочноямковий край Suprafoveal margin

Орбітальний край виличної кістки Orbital edge of zygomatic bone

Підочноямковий край Subfoveal edge

Ділянка надперенісся Section of the bridge of the nose

Ділянка лобного відростка верхньої щелепи Section of the frontal process of the upper jaw

140 / 200
При макроскопічному дослідженні зуба у пульпі визначається маса сіро-чорного кольору з гнильним, смердючим запахом. Мікроскопічно пульпа являє собою безструктурні зернисті маси з великою кількістю мікробів. Яка патологія розвинулась у пульпі зуба? During a macroscopic examination of the tooth in the pulp, a gray-black mass with a putrid, foul-smelling odor is determined. Microscopically, the pulp is a structureless granular mass with a large number of microbes. What pathology has developed in the tooth pulp?

Скупчення гемосидерину Hemosiderin accumulation

Гангрена Gangrene

Серозне запалення Serous inflammation

!нфаркт !heart attack

Гнійне запалення Purulent inflammation

141 / 200
У чоловіка 71-го року впродовж 10 днів спостерігався пронос з домішками у калі слизу та крові. Хворого було госпіталізовано у тяжкому стані, помер через 2 доби. При розтині тіла померлого виявлено: дифтеритичний коліт з множинними виразками неправильної форми різної глибини в сигмоподібній і прямій кишках. При бактеріологічному дослідженні висіяно шигели. Яке основне захворювання у хворого? A 71-year-old man had diarrhea with impurities of mucus and blood in his stool for 10 days. The patient was hospitalized in serious condition, died 2 days later. At autopsy the body of the deceased revealed: diphtheritic colitis with multiple ulcers of irregular shape of different depths in the sigmoid and rectum. Bacteriological examination revealed Shigella. What is the patient's main disease?

!єрсиніоз !yersiniosis

Неспецифічний виразковий коліт Nonspecific ulcerative colitis

Дизентерія Dysentery

Сальмонельоз Salmonellosis

Черевний тиф Typhoid

142 / 200
При мікроскопічному дослідженні біоптата з товстої кишки виявлена пухлина з призматичного епітелію, що формує атипові залозисті структури різної форми і величини. Базальна мембрана залоз зруйнована. Клітки пухлини поліморфні, ядра гіперхромні, відзначається велика кількість патологічних мітозів. Який діагноз найбільш вірогідний? During microscopic examination of a biopsy from the colon, a tumor of prismatic epithelium was found, which forms atypical glandular structures of various shapes and sizes. The basal membrane of the glands is destroyed. The tumor cells are polymorphic, the nuclei hyperchromic, a large number of pathological mitoses are noted. What is the most likely diagnosis?

Аденокарцинома Adenocarcinoma

Слизовий рак Mucosal cancer

Солідний рак Solid cancer

Базальноклітинний рак Basal cell carcinoma

Недиференційований рак Undifferentiated cancer

143 / 200
У пацієнта на слизовій оболонці ротової порожнини, носа та губах з’явились везикулярні пухирці. Стоматолог запідозрив везикулярний стоматит. Яке дослідження надасть можливість підтвердити діагноз? The patient has vesicular blisters on the mucous membrane of the oral cavity, nose, and lips. The dentist suspected vesicular stomatitis. What research would provide an opportunity to confirm the diagnosis?

Постановка алергічної проби Allergy testing

Зараження тварин везикулярною рідиною Infection of animals with vesicular fluid

Виділення вірусу з везикулярної рідини Isolation of virus from vesicular fluid

Мікроскопія везикулярної рідини Microscopy of vesicular fluid

Виділення бактерій з везикулярної рідини Isolation of bacteria from vesicular fluid

144 / 200
Під час операції на щелепно-лицьовій ділянці для зниження секреції слинних залоз хворому ввели атропіну сульфат. Виникли тахікардія, сухість та гіперемія шкіри, параліч акомодації та підвищення внутрішньоочного тиску. Який антагоніст доцільно використати в даному випадку? During an operation on the maxillofacial region, atropine sulfate was administered to the patient to reduce salivary gland secretion. Tachycardia, dryness and hyperemia of the skin, paralysis of accommodation, and increased intraocular pressure occurred. What antagonist should be used in this case?

Прозерин Proserin

Скополаміну гідробромід Scopolamine hydrobromide

Платифіліну гідротартрат Platiphylline hydrotartrate

Атенолол Atenolol

Дипіридамол Dipyridamole

145 / 200
У пацієнта 42-х років, що страждає на парадонтоз, у коронковій частині пульпи виявлено округлі звапновані утворення діаметром 2-3 мм. Назвіть ці структури: In a 42-year-old patient suffering from periodontitis, rounded calcified formations with a diameter of 2-3 mm were found in the crown part of the pulp. Name these structures:

Інтерглобулярні простори Interglobular spaces

Мертвий дентин Dead dentine

Дентиклі Denticles

Склерозований (прозорий) дентин Sclerosed (transparent) dentin

Зубні камені Dental stones

146 / 200
При остеолатеризмі зменшується міцність колагену, що зумовлена помітним зменшенням утворення поперечних зшивок у колагенових фібрилах. Причиною цього явища є зниження активності такого ферменту: With osteolaterism, the strength of collagen decreases, which is due to a noticeable decrease in the formation of cross-links in collagen fibrils. The reason for this phenomenon is a decrease in the activity of such an enzyme:

Моноаміноксидаза Monoamine oxidase

Лізілоксидаза Lysyl oxidase

Пролілгідроксилаза Prolylhydroxylase

Колагеназа Collagenase

Лізілгідроксилаза Lysyl hydroxylase

147 / 200
Для морфологічного дослідження представлена ендокринна залоза, паренхіма якої складається з єпітєлію та нервової тканини. В епітеліальних тра-бекулах виявляється 2 типи клітин: хро-мофільні та хромофобні. Визначте даний орган: An endocrine gland whose parenchyma consists of epithelium and nerve tissue is presented for morphological examination. 2 types of cells are found in epithelial trabeculae: chromophilic and chromophobic. Define this body:

Надниркова залоза Adrenal gland

Щитоподібна залоза Thyroid

Паращитоподібна залоза Parathyroid gland

Гіпоталамус Hypothalamus

Гіпофіз Pituitary

148 / 200
До приймального відділення надійшов хворий з ознаками гострої серцевої недостатності: блідість, акроціаноз, часте, поверхневе дихання. Який з перелічених засобів показаний у цьому випадку? A patient with signs of acute heart failure arrived at the reception department: paleness, acrocyanosis, frequent shallow breathing. Which of the listed remedies is indicated in this case?

Адреналіну гідрохлорид Adrenaline hydrochloride

Корглікон Corglycon

Кордіамін Cordiamine

Нітрогліцерин Nitroglycerin

Дигітоксин Digitoxin

149 / 200
У чоловіка 60-ти років спостерігається послаблення перистальтики кі-шечника. Який з перерахованих видів їжі буде стимулювати перистальтику в найбільшій мірі? A 60-year-old man has decreased peristalsis of the small intestine. Which of the listed types of food will stimulate peristalsis to the greatest extent?

М’ясо Meat

Білий хліб White bread

Чорний хліб Black bread

Чай Tea

Сало Lard

150 / 200
Під час цитогенетичного обстеження пацієнта з порушеною репродуктивною функцією виявлено в деяких клітинах нормальний каріотип 46,ХУ але у більшості клітин каріотип синдрому Клайнфельтера - 47,ХХУ Яку назву носить таке явище неоднорідності клітин? During the cytogenetic examination of a patient with impaired reproductive function, a normal karyotype of 46,XU was found in some cells, but in most cells the karyotype of Klinefelter syndrome is 47,XXU What is the name of this the phenomenon of cell heterogeneity?

Транспозиція Transposition

Мозаїцизм Mosaicism

Мономорфізм Monomorphism

Інверсія Inversion

Дуплікація Duplication

151 / 200
У чоловіка і його сина інтенсивно росте волосся по краю вушних раковин. Це явище спостерігалося також у батька і дідуся за батьківською лінією. Який тип успадкування зумовлює це? A man and his son have intensively growing hair on the edge of the earlobes. This phenomenon was also observed in the father and grandfather on the paternal line. What type of inheritance causes this?

Аутосомно-домінантний Autosomal dominant

Рецесивний, зчеплений з Х- хромосомою X-linked recessive

Аутосомно-рецесивний Autosomal recessive

Зчеплений з Y-хромосомою Linked to Y-chromosome

Домінантний, зчеплений з Х-хромосомою X-linked dominant

152 / 200
Сечокам’яна хвороба ускладнилася виходом конкременту з нирки. На якому рівні сечоводу, найімовірніше, він може зупинитися? The urolithiasis was complicated by the release of a stone from the kidney. At what level of the ureter is it most likely to stop?

У нирковій мисці In the renal pelvis

На межі черевної та тазової частин On the border of the abdominal and pelvic parts

На 5 см вище тазової частини 5 cm above the pelvis

На 2 см вище впадіння в сечовий міхур 2 cm above the confluence with the bladder

В середній черевній частині In the middle abdomen

153 / 200
В гістологічному препараті нижньої щелепи ембріону виявляється зубний зачаток, в якому зубний сосочок утворений дрібними зірчастими базо-фільно забарвленими клітинами. Яка тканина утворює цю частину зубного зачатку? In the histological preparation of the lower jaw of the embryo, a tooth bud is revealed, in which the dental papilla is formed by small stellate basophil-stained cells. What tissue forms this part of the tooth bud?

Епітеліальна Epithelial

Ретикулярна Reticular

Хрящова Khryaschova

Кісткова Bone

Мезенхіма Mesenchyme

154 / 200
Хворому 25-ти років з клінічною картиною нефротичного синдрому проведено пункційну біопсію нирки. Під час мікроскопічного дослідження клітини епітелію проксимальних канальців нефрону збільшені в об’ємі, у цитоплазмі вакуолі з прозорою рідиною, ядро зміщене до периферії. Яка дистрофія виявлена в епітелії каналь-ців? A 25-year-old patient with a clinical picture of nephrotic syndrome underwent a puncture biopsy of the kidney. During a microscopic examination, the cells of the epithelium of the proximal tubules of the nephron are enlarged in volume, in the cytoplasm of vacuoles with a clear liquid, the nucleus is shifted to the periphery. What kind of dystrophy is found in the epithelium of the tubules?

Жирова Fat

Гідропічна Hydropic

Зерниста Granular

Гіаліново-крапельна Hyaline-drip

Рогова Rohova

155 / 200
За професійними показаннями проведена вакцинація лікарів-стоматологів. Вакцина має захищати їх від вірусної інфекції, збудник якої може бути присутній у крові стоматологічних хворих, які перенесли інфекцію або є хронічними носіями. Яка вакцина була використана? According to professional indications, vaccination of dentists was carried out. The vaccine should protect them from a viral infection, the causative agent of which may be present in the blood of dental patients who have suffered an infection or are chronic carriers. What vaccine was used?

Генно-інженерний HBs-антиген Genetically engineered HBs-antigen

Антирабiчна вакцина Anti-rabies vaccine

Інактивована вакцина проти гепатиту А Inactivated hepatitis A vaccine

Жива корова вакцина Live cow vaccine

Субодинична грипозна вакцина Subunit influenza vaccine

156 / 200
У хворого виявлено порушення секреторної функції піднижньощелепної слинної залози. Який нерв забезпечує її вегетативну іннервацію? The patient has a violation of the secretory function of the submandibular salivary gland. What nerve provides its vegetative innervation?

Chorda tympani Chorda tympani

N. mandibularis N. mandibularis

N. petrosus major N. petrosus major

N. petrosus minor N. petrosus minor

N. auriculotemporalis N. auriculotemporalis

157 / 200
До лабораторії особливо небезпечних інфекцій доставлено матеріал хворого з підозрою на холеру. Який метод експрес діагностики може підтвердити цей діагноз? The material of a patient suspected of cholera was delivered to the laboratory of especially dangerous infections. What method of rapid diagnosis can confirm this diagnosis?

Р!Ф Р!Ф

PЗК PZK

РГА РГА

РА RA

РП RP

158 / 200
Хворому на остєомієліт нижньої щелепи був призначений антибіотик тетрациклінової групи. Вкажіть цей препарат: A tetracycline antibiotic was prescribed to a patient with osteomyelitis of the lower jaw. Specify this drug:

Рифампіцин Rifampicin

Стрептоміцин Streptomycin

Доксицикліну гідрохлорид Doxycycline hydrochloride

Амікацин Amikacin

Оксацилін Oxacillin

159 / 200
При обстеженні хворого похилого віку після тромбозу судин головного мозку виявлено моторну афазію. Де локалізований осередок пошкодження? During the examination of an elderly patient after thrombosis of cerebral vessels, motor aphasia was detected. Where is the localized focus of damage?

Центр Верніке Wernicke Center

Центр Брока Broca Center

Прецентральна звивина Precentral gyrus

Звивина Гешля Heschl's gyrus

Кутова звивина Angular gyrus

160 / 200
При аналізі ЕКГ пацієнта встановлено збільшення тривалості зубця P. З чим це пов’язано? During the analysis of the patient's ECG, an increase in the duration of the P wave was detected. What is this connected with?

Уповільнене розповсюдження збудження шлуночками Delayed ventricular excitation propagation

Прискорене розповсюдження збудження шлуночками Accelerated ventricular excitation propagation

Прискорене проведення збудження через атріовентрикулярний вузол Accelerated conduction of excitation through the atrioventricular node

Уповільнене розповсюдження збудження передсердями Delayed propagation of atrial excitation

Прискорене розповсюдження збудження передсердями Accelerated atrial excitation propagation

161 / 200
У новонародженої дитини виявлено вроджені вади розвитку травної системи, що пов’язано з дією тератоген-них факторів на початку вагітності. На який зародковий листок подіяв терато-ген? Congenital malformations of the digestive system were detected in a newborn child, which is associated with the action of teratogenic factors at the beginning of pregnancy. Which embryonic sheet was affected by the teratogen?

Ектодерма Ectoderm

Ендодерма Endoderm

Мезодерма Mesoderm

Ендодерма і мезодерма Endoderm and Mesoderm

Усі листки All leaves

162 / 200
Жінка під час вагітності хворіла на краснуху. Дитина народилась з вадами розвитку - незрощення губи і піднебіння. Генотип у дитини нормальний. Ці аномалії розвитку є проявом: The woman had rubella during pregnancy. The child was born with malformations of the lip and palate. The child's genotype is normal. These developmental abnormalities are a manifestation of:

Комбінативної мінливості Combinative variability

Модифікаційної мінливості Modification variability

Хромосомної мутації Chromosomal mutation

Анеуплодії Aneuplodia

Поліплоїдії Polyploids

163 / 200
Хворому 67-ми років з діагнозом: ішемічна хвороба серця, стенокардія напруги, був призначений лікарський препарат з групи блокаторів калієвих каналів. Який препарат був призначений хворому? A 67-year-old patient with a diagnosis of coronary heart disease, angina pectoris was prescribed a drug from the group of potassium channel blockers. What drug was prescribed to the patient?

Карбокромен Carbochromene

Дипіридамол Dipyridamole

Молсидомін Molsidomin

Амлодипін Amlodipine

Аміодарон Amiodarone

164 / 200
У жінки, що тривало приймала антибіотики з приводу кишкової інфекції, розвинулося ускладнення з боку слизової порожнини рота у вигляді запального процесу і білого нальоту, у якому при бактеріологічному дослідженні були виявлені дріжджеподібні грибки Candida albicans. Який з перерахованих препаратів показаний для лікування цього ускладнення? A woman who had been taking antibiotics for a long time due to an intestinal infection developed complications from the mucous membrane of the mouth in the form of an inflammatory process and white plaque, in which bacteriological examination showed yeast-like fungi Candida albicans were detected. Which of the listed drugs is indicated for the treatment of this complication?

Фуразолідон Furazolidone

Бісептол Biseptol

Поліміксин Polymixin

Флуконазол Fluconazole

Тетрациклін Tetracycline

165 / 200
У хворої після сильного психоемоційного навантаження розвинувся неспецифічний патологічний процес, що передбачає 3 стадії: реакцію тривоги, стадію резистентності, стадію виснаження. Як він називається? After a strong psycho-emotional stress, the patient developed a non-specific pathological process, which involves 3 stages: anxiety reaction, resistance stage, exhaustion stage. What is it called?

Регенерація Regeneration

Компенсація Compensation

Парабіоз Parabiosis

Стрес Stress

Адаптація Adaptation

166 / 200
Людина вийшла з кондиціоновано-го приміщення назовні, де немає вітру, температура повітря +38oC, вологість 64%. За рахунок якого механізму буде здійснюватися віддача тепла з організму за цих умов? A person left the air-conditioned room outside, where there is no wind, the air temperature is +38oC, the humidity is 64%. At the expense of which mechanism will the heat transfer from the body be carried out according these conditions?

Радіація Radiation

Випаровування поту Evaporation of sweat

Конвекція Convection

Проведення Performance

Кондукція Conduction

167 / 200
У хворого виражений больовий синдром при невралгії. Який засіб з не-стероїдних протизапальних препаратів зменшить болесприйняття? The patient has a pronounced pain syndrome with neuralgia. Which non-steroidal anti-inflammatory drug will reduce pain perception?

Кетаміну гідрохлорид Ketamine hydrochloride

Лідокаїну гідрохлорид Lidocaine hydrochloride

Кодеїну фосфат Codeine Phosphate

Дроперидол Droperidol

Диклофенак-натрій Diclofenac sodium

168 / 200
Пацієнта на дачі вжалила бджола. Об’єктивно: кисть лівої руки гаряча, рожева, набрякла, у місці укусу великий червоний піхур. Який механізм є провідним у розвитку набряку? The patient was stung by a bee in the country. Objectively: the hand of the left hand is hot, pink, swollen, there is a large red blister at the site of the bite. What is the leading mechanism in the development of edema ?

Пошкодження судин при жаленні Damage of blood vessels during stinging

Зниження онкотичного тиску тканинної рідини Reduction of oncotic pressure of tissue fluid

Підвищення проникливості судин Increasing vascular permeability

Зниження осмотичного тиску тканинної рідини Decrease in tissue fluid osmotic pressure

Зниження кровонаповнення судин Decreased blood vessel filling

169 / 200
У пацієнта 32-х років має місце гіповітаміноз B2. Причиною виникнення специфічних симптомів (ураження епітелію, слизових, шкіри, рогівки ока) найбільш імовірно є дефіцит: A 32-year-old patient has hypovitaminosis B2. The cause of specific symptoms (epithelial, mucous, skin, corneal lesions) is most likely a deficiency:

Цитохрому с Cytochrome c

Цитохрому а1 Cytochrome a1

Цитохрому в Cytochrome in

Цитохромоксидази Cytochrome oxidases

Флавінових коферментів Flavin coenzymes

170 / 200
У хворого при кашлі виділяється іржаво-коричневе харкотиння, в якому виявляються овальні, золотаво-коричневі яйця розміром близько 0,1 мм. Який діагноз може бути поставлений у даному випадку? The patient coughs up rusty-brown sputum, in which oval, golden-brown eggs about 0.1 mm in size are found. What diagnosis can be made in this case?

Дикроцеліоз Dicroceliosis

Шистосомоз Schistosomiasis

Фасціольоз Fasciolosis

Опісторхоз Opistorchosis

Парагонімоз Paragonimosis

171 / 200
У пробірку, що містить 0,3% розчин NaCl, додали краплю крові. Що відбудеться з еритроцитами? A drop of blood was added to a test tube containing 0.3% NaCl solution. What will happen to the erythrocytes?

Біологічний гемоліз Biological hemolysis

Механічний гемоліз Mechanical hemolysis

Осмотичний гемоліз Osmotic hemolysis

Зморшкування Wrinkle

Змін не буде There will be no changes

172 / 200
При гістологічному дослідженні мікропрепарату шкіри людини виявляється тільки щільна неоформлена сполучна тканина. Який шар даного органу був представлений для вивчення? During histological examination of a micropreparation of human skin, only dense, irregular connective tissue is detected. What layer of this organ was presented for study?

Підшкірна жирова клітковина (гіподерма) Subcutaneous adipose tissue (hypodermis)

Базальний шар епідермісу Basal layer of the epidermis

Сітчастий шар дерми Reticular layer of the dermis

Сосочковий шар дерми Papillary layer of the dermis

Епідерміс Epidermis

173 / 200
Після перенесеної травми голови у хворого бувають напади інтенсивного болю в ділянці обличчя і судоми жувальних м’язів. Який нерв найімовірніше травмований? After a head injury, the patient has attacks of intense pain in the face and spasms of the masticatory muscles. Which nerve is most likely injured?

N. oculomotorius N. oculomotorius

N. trigeminus N. trigeminus

N. abducens N. abducens

N. facialis N. facialis

N. olphactorius N. olphactorius

174 / 200
Після обстеження хворого лікар рекомендував йому вилучити з раціону наваристі м’ясні та овочеві бульйони, прянощі, копчені продукти, оскільки у хворого було виявлено: After examining the patient, the doctor recommended that he remove rich meat and vegetable broths, spices, and smoked products from his diet, as the patient was found to have:

Зменшення моторики шлунково-кишкового тракту Decreased motility of the gastrointestinal tract

Дискінєзія жовчних шляхів Biliary dyskinesia

Збільшення секреції хлористоводневої кислоти залозами шлунка Increased secretion of hydrochloric acid by the gastric glands

Зменшення секреції хлористоводневої кислоти залозами шлунка Decreased secretion of hydrochloric acid by the gastric glands

Зменшення слиновиділення Decreased salivation

175 / 200
Виявлення здатності до капсу-лоутворення у багатьох збудників є важливим при проведенні експрес-діагностики та складовим бактеріологічного дослідження. Для виявлення капсули досліджуваної культури в умовах лабораторії можна застосовувати фарбування за таким простим методом, як: Detecting the ability to form a capsule in many pathogens is important for rapid diagnostics and a component of bacteriological research. To detect the capsule of the studied culture in laboratory conditions, staining according to by such a simple method as:

Буррі Burri

Буррі-Гінса Burri-Guinsa

Дроботько Drobotko

Лєффлера Loeffler

Романовського-Гімзи Romanovsky-Giemza

176 / 200
Лікар призначив хворому з гострою серцевою недостатністю не-глікозидний кардіотонічний засіб, який безпосередньо стимулює ві -адренорецептори міокарда, збільшує кровообіг, діурез та застосовується лише внутрішньовенно крапельно внаслідок швидкої інактивації в організмі. Який препарат призначив лікар? The doctor prescribed a non-glycoside cardiotonic agent to a patient with acute heart failure, which directly stimulates v-adrenoceptors of the myocardium, increases blood circulation, diuresis and is used only intravenously as a drip due to rapid inactivation in the body. What drug did the doctor prescribe?

Адреналін Adrenaline

Добутамін Dobutamine

Дигоксин Digoxin

Анаприлін Anaprilin

Корглікон Corglycon

177 / 200
У дитини 9-ти років, що хворіла на грип, на 5-ту добу захворювання з’явилися сильний головний біль, нудота, запаморочення, менінгеальні знаки. Смерть настала через добу від набряку мозку, що наростав. Під час розтину порожнини черепа м’які мозкові оболонки набряклі, повнокровні, дифузно просякнуті яскраво-червоного кольору рідиною. Про яке ускладнення грипу слід думати? A 9-year-old child who had the flu developed a severe headache, nausea, dizziness, and meningeal signs on the 5th day of the illness. Death came a day later from swelling of the brain, which was growing. During the dissection of the skull cavity, the soft meninges are swollen, full of blood, diffusely soaked with a bright red liquid. What complication of influenza should be considered?

Венозна гіперемія оболонок мозку Venous hyperemia of the meninges

Геморагічний менінгіт Hemorrhagic meningitis

Крововилив в мозок Brain hemorrhage

Гнійний лептоменінгіт Suppurative leptomeningitis

Серозний менінгіт Serous meningitis

178 / 200
У дитини 3-х років мати звернула увагу на утруднене дихання і слизові виділення з носа, що з’явилися після контакту з квітами. При біопсійному дослідженні у слизовій оболонці носа вияв- лено набряк і наявність великої кількості еозинофільних лейкоцитів. Який патологічний стан розвинувся у дитини? In a 3-year-old child, the mother drew attention to difficult breathing and mucous secretions from the nose that appeared after contact with flowers. During a biopsy examination in the mucous membrane swelling of the nose and the presence of a large number of eosinophilic leukocytes were detected. What pathological condition has developed in the child?

Недостатність імунної системи Insufficiency of the immune system

Клітинно-опосередкована реакція Cell-mediated response

Атопічна реакція Atopic reaction

Антитільна реакція Antibody reaction

Імунокомплексна реакція Immunocomplex reaction

179 / 200
На розтині тіла чоловіка 47-ми років, що помер від легеневої кровотечі, у 2 сегменті правої легені виявлена порожнина округлої форми з нерівними краями розміром 5,5 см; внутрішня поверхня її вкрита щільнуватими жовтуватими масами, що переходять у тканину легені. При гістологічному дослідженні - внутрішній шар складається з розплавлених казеозних мас, некро-тизованої тканини легені. Який процес розвинувся в легені? At the autopsy of a 47-year-old man who died of pulmonary hemorrhage, a 5.5 cm round cavity with uneven edges was found in the 2nd segment of the right lung; its inner surface is covered with dense yellowish masses that pass into the lung tissue. During histological examination, the inner layer consists of melted caseous masses, necrotic lung tissue. What process developed in the lung?

Рак легені, що розпадається Decomposing lung cancer

Гострий абсцес Acute abscess

Хронічний абсцес Chronic abscess

Хронічний кавернозний туберкульоз Chronic cavernous tuberculosis

Гострий кавернозний туберкульоз Acute cavernous tuberculosis

180 / 200
Циклічні зміни слизової оболонки матки обумовлені дією гормонів яєчника на судини матки. Атрофія жовтого тіла, яке продукує гормон прогестерон, призводить до спазму судин. Які це судини? Cyclic changes of the uterine mucosa are due to the action of ovarian hormones on the uterine vessels. Atrophy of the corpus luteum, which produces the hormone progesterone, leads to spasm of the vessels. What are these vessels?

Судини надсудинного шару міоме-трію Vessels of the supravascular layer of the myometrium

Прямі артерії ендометрію Direct arteries of the endometrium

Судини судинного шару міометрію Vessels of the vascular layer of the myometrium

Судини підслизового шару міометрію Vessels of the submucosal layer of the myometrium

Спіралеподібні артерії ендометрію Spiral arteries of the endometrium

181 / 200
Хворий повернувся із Африки з високою температурою (39oC), жовтяницею. Стан погіршувався, хворий помер. На розтині тіла виявлена жовтяниця, збільшені в розмірі аспидно-сірі печінка та селезінка, наявність гемомелані-ну в кістковому мозку. Реакція Перлса позитивна. Визначте імовірну причину смерті: The patient returned from Africa with a high temperature (39oC), jaundice. The condition worsened, the patient died. An autopsy revealed jaundice, enlarged asp-gray liver and spleen, the presence of hemelanin in the bone marrow. The Perls reaction is positive. Determine the probable cause of death:

Малярія Malaria

Амебіаз Amebiasis

Опісторхоз Opistorchosis

Бластомікоз Blastomycosis

Ехінококоз Echinococcosis

182 / 200
У хворого на пієлонефрит сеча тимчасово втратила бактерицидність через пошкодження клітин, які знахо- дяться у збиральних трубочках нирок та забезпечують секреторну фазу се-чоутворення. Які це клітини? In a patient with pyelonephritis, the urine temporarily lost its bactericidal properties due to damage to the cells located in the collecting tubules of the kidneys and providing the secretory phase of urine formation. What are these cells?

Гладенькі міоцити Smooth myocytes

Темні клітини Dark cells

Облямовані епітеліоцити Bordered epitheliocytes

Світлі клітини Light cells

Пласкі епітеліоцити Squamous epitheliocytes

183 / 200
Хворий 37-ми років доставлений до лікарні у тяжкому стані. Об’єктивно: виражена набряклість язика, гортані, губ, повік. Дихання утруднене. Обличчя ціанотичне. Відомо, що 2 години тому лікував зуби у стоматолога. Про який алергічний стан йдеться? A 37-year-old patient was brought to the hospital in serious condition. Objectively: severe swelling of the tongue, larynx, lips, eyelids. Breathing is difficult. The face is cyanotic. It is known , that 2 hours ago he had his teeth treated at the dentist. What allergic condition is it about?

Бронхіальна астма Bronchial asthma

Набряк Квінке Quincke edema

Кропив’янка Hives

Сезонний риніт Seasonal rhinitis

Феномен Артюса Arthus Phenomenon

184 / 200
Хворий звернувся до лікаря зі скаргами на періодичні висипання герпети-чних пухирців на лінії губ і на крилах носа. Такий стан спостерігається впродовж 10-ти років, кожний раз після зниження захисних сил організму. Лікар встановив діагноз: лабіальний герпес. Як називається така форма інфекції? The patient turned to the doctor with complaints of periodic eruptions of herpetic blisters on the line of the lips and on the wings of the nose. This condition has been observed for 10 years, each time after a decrease in the body's defenses. The doctor made a diagnosis: labial herpes. What is this form of infection called?

Екзогенна Exogenous

Гостра Acute

Затяжна Prolonged

Латентна Latent

Персистенція Persistence

185 / 200
Для зменшення зубного болю хворому рекомендували приймати ненар-котичний анальгетик. Який засіб був призначений? To reduce toothache, the patient was recommended to take a non-narcotic analgesic. What remedy was prescribed?

Індерал Inderal

Анестезин Anesthesia

Метацин Metacin

Фізостигміну саліцилат Physostigmine salicylate

Парацетамол Paracetamol

186 / 200
Хворому, що страждає на алергічний хейліт, призначена мазь преднізолону для змазування червоної облямівки і слизової губ. До якої групи засобів відноситься цей препарат? A patient suffering from allergic cheilitis is prescribed prednisone ointment to lubricate the red border and mucous membrane of the lips. What group of drugs does this drug belong to?

Глюкокортикоїди Glucocorticoids

Антагоністи лейкотрієнових рецепторів Leukotriene receptor antagonists

Блокатори серотонінових рецепторів Blockers of serotonin receptors

Блокатори гістамінових рецепторів Blockers of histamine receptors

Мембраностабілізатори Membrane stabilizers

187 / 200
У дівчинки 8-ми років раптово підвищилась температура, з’явились катаральні явища дихальних шляхів. На 5-й день хвороби виник параліч м’язів нижніх кінцівок і приєднались дихальні розлади. В передніх рогах спинного мозку виявлена проліферація глії навколо загиблих нейронів. Про яке захворювання йдеться? An 8-year-old girl had a sudden rise in temperature, catarrhal symptoms of the respiratory tract appeared. On the 5th day of the illness, paralysis of the muscles of the lower limbs occurred and joined respiratory disorders. In the anterior horns of the spinal cord, the proliferation of glia around the dead neurons was detected. What disease is it about?

Кір Measles

Поліомієліт Polio

Менінгококцемія Meningococcemia

Скарлатина Scarlatina

Дифтерія Diphtheria

188 / 200
Хворий 52-х років зі стійким підвищенням артеріального тиску до 200/110 мм рт.ст. помер при явищах хронічної серцево-судинної недостатності. На розтині: маса серця 600 г, товщина стінки лівого шлуночка 2,2 см, порожнини серця розширені. Гістологічно виражений гіаліноз і склероз артеріол міокарда. Для якого захворювання характерні описані зміни? A 52-year-old patient with a persistent increase in blood pressure to 200/110 mm Hg died of chronic cardiovascular failure. At autopsy: heart mass 600 g, the thickness of the wall of the left ventricle is 2.2 cm, the cavities of the heart are enlarged. Histologically pronounced hyalinosis and sclerosis of the arterioles of the myocardium. For which disease are the described changes characteristic?

Дилятаційна кардіоміопатія Dilated cardiomyopathy

Ендоміокардіальний фіброз Endomyocardial fibrosis

Атеросклероз артерій серця Atherosclerosis of the arteries of the heart

Гіпертонічна хвороба Hypertensive disease

Гіпертрофічна кардіоміопатія Hypertrophic cardiomyopathy

189 / 200
У хворого під час гіпертонічного кризу виник геморагічний інсульт, внаслідок чого спостерігається відсутність довільних рухів, підвищення сухожильних рефлексів та тонусу м’язів лівих руки та ноги. Як називається таке порушення рухової функції? The patient had a hemorrhagic stroke during a hypertensive crisis, as a result of which there was a lack of voluntary movements, increased tendon reflexes and muscle tone of the left arm and leg. What is this called violation of motor function?

Млявий параліч Lethargic paralysis

Тетраплегія Tetraplegia

Моноплегія Monoplegia

Геміплегія Hemiplegia

Параплегія Paraplegia

190 / 200
Дитині 5-ти років був діагностований гінгівіт. Лікар-стоматолог обробив місця ураження галогеновим антисептиком, який має протимікробну, фунгіцидну дію. Який це лікарський засіб? A 5-year-old child was diagnosed with gingivitis. The dentist treated the affected areas with a halogen antiseptic, which has an antimicrobial, fungicidal effect. What is this medicine?

Розчин фуразолідону Furazolidone solution

Розчин цинку сульфату Solution of zinc sulfate

Розчин йоду спиртовий Iodine alcohol solution

Протаргол Protargol

Розчин кислоти борної Boric acid solution

191 / 200
При лікуванні хворого на спадкову форму імунодефіциту було застосовано метод генотерапії: ген ферменту був внесений у клітини пацієнта за до- помогою ретровірусу. Яка властивість генетичного коду дозволяє використовувати ретровіруси у якості векторів функціональних гєнів? In the treatment of a patient with a hereditary form of immunodeficiency, the method of gene therapy was used: the enzyme gene was introduced into the patient's cells with the help of a retrovirus. What property of the genetic code allows the use of retroviruses in qualities of vectors of functional genes?

Специфічність Specificity

Універсальність Versatility

Колінеарність Colinearity

Надмiрнiсть Excess

Бєзпєрєрвність Continuity

192 / 200
Скєлєтні м’язи прикріплюються до кісток за допомогою сухожиль, які здатні витримувати велике силове навантаження. Яким видом сполучної тканини утворені сухожилля? Skeletal muscles are attached to bones with the help of tendons, which are able to withstand a large force load. What type of connective tissue are tendons made of?

Пухка волокниста Fluffy fibrous

Ретикулярна Reticular

Щільна оформлена Dense Formatted

Хрящова Khryaschova

Щільна неоформлена Dense unformatted

193 / 200
Хворий звернувся з відчуттям серцебиття після стресу. ЧСС- 104/хв, тривалість інтервалу P — Q - 0,12 сек, QRS - без змін. Який тип аритмії у хворого? The patient complained of palpitations after stress. Heart rate - 104/min, duration of the P — Q interval - 0.12 sec, QRS - unchanged. What type of arrhythmia in the patient?

Синусова тахікардія Sinus tachycardia

Миготлива аритмія Atrial fibrillation

Синусова аритмія Sinus arrhythmia

Синусова брадикардія Sinus bradycardia

Екстрасистолія Extrasystole

194 / 200
При огляді порожнини рота лікар виявив каріозну порожнину на поверхні першого малого кутнього зуба, зверненої до ікла. Як називається уражена поверхня коронки? During the examination of the oral cavity, the doctor found a carious cavity on the surface of the first small canine tooth facing the canine. What is the affected surface of the crown called?

Facies distalis Facies distalis

Facies lingualis Facies lingualis

Facies vestibularis Facies vestibularis

Facies occlusalis Facies occlusalis

Facies mesialis Facies mesialis

195 / 200
У дитини 6-ти років спостерігається затримка росту, порушення процесів окостеніння, декальцифікація зубів. Що може бути причиною цього? A 6-year-old child has growth retardation, disruption of ossification processes, decalcification of teeth. What could be the reason for this?

Гіпертиреоз Hyperthyroidism

Авітаміноз С Avitaminosis C

Інсулінова недостатність Insulin deficiency

Дефіцит вітаміну D Vitamin D deficiency

Зменшення продукції глюкагону Decreased glucagon production

196 / 200
У хворого з алкогольним ураженням печінки порушені процеси біо-трансформації ксенобіотиків та ендогенних токсичних сполук. Зниження активності якого хромопротеїну може бути причиною цього? In a patient with alcoholic liver damage, the processes of bio-transformation of xenobiotics and endogenous toxic compounds are disturbed. A decrease in the activity of which chromoprotein can be the cause of this?

Цитохромоксидаза Cytochrome oxidase

Цитохром b Cytochrome b

Цитохром с1 Cytochrome c1

Гемоглобін Hemoglobin

Цитохром Р-450 Cytochrome P-450

197 / 200
Під час ембріонального розвитку зубів у деяких клітин, що продукують структури зуба, має місце інверсія ядер та органел з базального полюсу клітини до апікального. Яку назву мають ці клітини? During the embryonic development of teeth, some cells that produce tooth structures undergo an inversion of nuclei and organelles from the basal pole of the cell to the apical pole. What are these cells called?

Енамелобласти Enameloblasts

Фібробласти Fibroblasts

Дентинобласти Dentinoblasts

Мезенхімні клітини Mesenchymal cells

Цементобласти Cement areas

198 / 200
На мікроскопічному препараті бронхів виявлено епітеліальний пласт одношарового багаторядного війчастого епітелію, в якому спостерігаються келихоподібні клітини. Яка їх функція? An epithelial layer of a single-layer multi-rowed ciliated epithelium was found on a microscopic specimen of the bronchi, in which goblet cells are observed. What is their function?

Скоротлива Abbreviated

Камбіальна Cambial

Опорна Reference

Всмоктувальна Suction

Залозиста Glandular

199 / 200
На розтині тіла жінки, що тривало страждала на аденому паращитоподі-бних залоз із гіперпродукцією парат-гормону та померла від хронічної ниркової недостатності, виявлені відкладання вапна у шлунку, легенях, нирках. Яке за механізмом розвитку звапнуван-ня має місце? At the autopsy of a woman who suffered from adenoma of the parathyroid glands with hyperproduction of parathyroid hormone and died of chronic kidney failure, lime deposits were found in the stomach, lungs , kidneys. What is the mechanism of development of calcification?

Змішане Mixed

Метаболічне Metabolic

- -

Дистрофічне Dystrophic

Метастатичне Metastatic

200 / 200
Від хворого виділені нерухомі бактерії овоїдної форми з біполярним забарвленням. В організмі утворюють ніжну капсулу. На агарі утворюють колонії з мутно-білим центром, оточеним фестончатою каймою, що нагадують мереживо. Продукують 'мишачий токсин'. Дані властивості притаманні для збудника: Immobile, ovoid-shaped bacteria with a bipolar color were isolated from the patient. They form a delicate capsule in the body. On agar, they form colonies with a cloudy white center surrounded by a scalloped border, resembling lace. They produce 'mouse toxin'. These properties are characteristic of the causative agent:

Чуми Plagues

Туляремії Tularemia

Бруцельозу Brucellosis

Коклюшу Whooping cough

Сибірки Anthrax